Sie sind auf Seite 1von 48

APPENDIX B

Chapter Reviews Answer Key


CHAPTER 1 ANIMAL ANATOMY AND PHYSIOLOGY
1. (Answer: b). Only mitochondria (b) are specifically identified as being present in large numbers in
cells that use large amounts of energy (e.g., skeletal muscle). Mitochondria are called the
powerhouse cells. The Golgi complex (a) modifies substances received from the endoplasmic
reticulum and produces the lysosomes (c), which digest enzymes and are the principal organelles in
digestion of nutrients. Thus they are found in large numbers in phagocytic cells, not MS cells. Finally
SER (d) is found in large amounts in liver, intestinal, and interstitial cells.
2. (Answer: c). Active transport is the movement of molecules from a low concentration to a high
concentration with the aid of carrier proteins, such as the sodium-potassium pump (c) that moves ions
and amino acids into cells by this method. By contrast, diffusion (a), osmosis (b), and facilitated
diffusion (d) are all examples of passive processes.
3. (Answer: a). Articular cartilage is hyaline cartilage (a) that covers the ends of the bones. The pinna (b)
contains elastic cartilage, and the pubic symphysis (d) contains fibrocartilage. Ligaments (c) are made
up of dense elastic connective tissue.
4. (Answer: d). Caudal (d) means toward the tail. The stomach is closer to the tail than the heart. Distal
(a) refers to the point farthest from the backbone. This term is used especially in reference to limbs.
Rostral (b) means toward the nose and is used to describe structures on the head. Finally, cranial (c)
means toward the head.
5. (Answer: a). Lacunae (a) are small spaces in compact bone that house osteocytes. The other structures
listed are also parts of compact bone. Haversian canals (b) house blood vessels and nerves; lamellae
(c) are the concentric rings of a compact bone; and canaliculi (d) are very small canals that radiate out,
connecting all lacunae to each other and to the central haversian canal.
6. (Answer: c). Cardiac muscle cells (c) are striated, have a single, centrally located nucleus, and are
joined by intercalated discs, which aid in conduction of the nervous impulse to coordinate
contraction. Flat bones (a) have two thin layers of compact bone with spongy bone between the layers.
Smooth muscle cells (b) have no striations, and, finally, neurons (d) are composed of dendrites, cell
body, and axon.
7. (Answer: c). Schwann cells are supportive and protective (c), wrapping around axons to form
protective myelin in peripheral nervesnot in the CNS (a). They are part of the PNS, not the
immune system (b), and the nodes of Ranvier (d) refers to the nodes that interrupt myelin, not form it,
to transmit impulses along myelinated nerve fibers.
8. (Answer: c). Only the pulmonary vein (c) returns blood from the lungs to the heart for distribution to
the systemic circulatory system. Thus the pulmonary vein is the only vessel in the body that carries
oxygenated blood, and not the vena cava, jugular vein, or pulmonary artery (a, b, and d).
9. (Answer: d). During ventricular systole, the electrical impulse that coordinates the heartbeat moves
(d) from the SA, to the AV node, which conducts the impulse down the bundle of His to the Purkinje
fibers. The other sequences are incorrect (a, b, and d).
10. (Answer: a). The P wave represents atrial depolarization (systole) (a). The QRS complex represents
depolarization of the ventricles (systole) (b). The T wave represents repolarization (diastole) of the
ventricles. (d) Repolarization (diastole) of the atria (c) occurs during the depolarization of the
ventricles and is masked by the QRS complex.
11. (Answer: d). The rumen is the most cranial compartment and is where food is mixed and churned,
followed by the reticulum for churning; the omasum grinds and absorbs water and bicarbonate, and
finally the abomasum, the true glandular stomach, initiates chemical digestion and connects to the
small intestine (d). The other choices are out of sequence (a, b, and c).
12. (Answer: a). Hydrochloric acid is a digestive enzyme. Only parietal and chief cells are found in the
digestive system. Parietal cells (a) produce hydrochloric acid, whereas the chief cells (b) produce
pepsinogen. Schwann cells (c) are part of the nervous system, and melanocytes (d) are found in the
integument system.
13. (Answer: c). During inspiration the diaphragm moves caudally (c)that is, toward the tail, while at
the same time the chest moves ventrally (a), increasing the size of the thoracic cavity. The other two
directions (b and d) are not a large part of inspiratory process.
14. (Answer: b). The loop of Henle (b) is found in the medulla, not the renal cortex. The outer cortex
contains the glomerulus, Bowmans capsule (a), proximal convoluted tubules (PCTs) (c), and distal
convoluted tubules (DCTs) (d).
15. (Answer: a). Epinephrine (a) stimulates the fight-or-flight reaction in the sympathetic nervous
system. Calcitonin (b) causes calcium to be stored in bone, decreasing blood levels; thyroxin (c)
accelerates metabolism; and prolactin (d) stimulates lactation.
16. (Answer: c). The iris (c) is the colored, contractile membrane between the lens and the cornea (d),
which is the transparent covering on the eye; it regulates the amount of light passing through the
pupil (a), the hole in the iris through which light passes to enter the posterior chamber. The lens (b)
focuses light onto the retina.
17. (Answer: c). The three auditory ossicles (malleus, incus, and stapes) are housed in the middle ear (c).
They transmit sound from the outer ear (b) (from the pinna to the tympanic membrane) to the inner
ear (d) (cochlea [a] and semicircular canals).
18. (Answer: a). Vasopressin is produced in the pituitary gland (a). Epinephrine and norepinephrine are
produced in the adrenal medulla (b); glucocorticoids, mineralocorticoids, and gonadocorticoids are
produced in the adrenal cortex (c). The spleen (d) filters, stores, and releases blood. It has no endocrine
function.
19. (Answer: c). Dogs (c) have a prostate only. Cats and horses (a and d) have both a bulbourethral gland
and a prostate. A cow (b) is female and thus has no male reproductive glands.
20. (Answer: c). The stratum granulosum in the epidermis (c) consists of two to four layers of flattened
keratocytes that help waterproof the skin. The stratum corneum (a) is the most superficial,
nonvascular layer that is constantly being shed, and the stratum germinativum (b) is the deepest layer
of the epidermis and contains pigment cells. The corium (d), or dermis, is the layer beneath the
epidermis and contains arteries, veins, capillaries, lymphatics, and nerve fibers.

CHAPTER 2 URINALYSIS AND HEMATOLOGY


1. (Answer: b). In both qualitative and semi-quantitative urine analysis, phenolsulfonphthalein (b) dye
is exogenous. Endogenous substances include uric acid, amino acids, hormones (a, c, and d) and
electrolytes.
2. (Answer: b). Samples should be analyzed within 20 to 30 minutes of collection (b) to maximize
validity of information and minimize postcollection analytical variables. It is not necessary (and
would be very difficult) to complete an analysis within 2 minutes of collection (a), and anything past
20 to 30 minutes (c and d) affects results.
3. (Answer: d). Normal freshly voided urine in many species is clear; exceptions include all three of the
animals listed here (d). Rabbits (a) and hamsters (c) may have normal but cloudy urine because of the
presence of calcium salts and the horse (b) because of the presence of calcium carbonate crystals and
mucus secreted by glands in the renal pelvis.
4. (Answer: b). At least 5 mL (b) fresh urine is ideal, but micro methods and containers are
available for smaller samplesalthough 1 mL (a) is not ideal. Both 10 and 20 mL (c and d) exceed the
ideal sample size.
5. (Answer: d). A 3- to 6-hour postprandial sample (d) may be more reflective of the diet. Anything less
than that (a, b, and c) will be less representative.
6. (Answer: a). Heparin (a) is the preferred anticoagulant when conducting hematological examinations
in both birds and reptiles. EDTA (b) is the best anticoagulant for hematological testing in most
mammals. Potassium chloride (c) is used to prevent or treat low blood levels of potassium
(hypokalemia), and ACD (d), though an anticoagulant is not the preferred agent in this case.
7. (Answer: a). When using anticoagulants, fill the tube to at least 90% of its capacity (a) to maintain the
proper anticoagulant-to-blood ratio. Anything less (b, c, and d) will make achieving the proper ratio
difficult.
8. (Answer: c). A blood sample collected immediately postprandial may be lipemic (c). It does not
accurately reflect icterus (a), a rise in TPP (b), or a drop in RBCs (d).
9. (Answer: b). Collection through cystocentesis (b) avoids contaminants from the lower portions of the
urinary tract, such as samples from manual expression (a), midstream catch (c), or litter pan pour-off
(d), making the sample suitable for bacterial culture.
10. (Answer: c). Pollakiuria refers to frequent urination (c), although it is often confused with polyuria,
which is excretion of large volumes of urine (b). Complete absence of urine formation (a) is anuria, and
decreased urine formation or elimination (d) is oliguria.
11. (Answer: c). A basket or smudge cell is a damaged leukocyte with a basket weave pattern and
nuclear remnant (c) that has been damaged from an overwhelming pathological process, in vitro
handling, excess anticoagulation, or extended period from collection to processing. It is not an
erythrocyte (a and b), nor is it a granular tissue cell (d).
12. (Answer: c). MCHC is calculated by multiplying the Hb concentration by 100 and dividing the
product by the PCV (c). Multiplying Hb by 10 and dividing the product by total RBC count (b) is the
formula for determining MCH, not MCHC. Multiplying PCV by 10 and dividing the product by total
RBC count (d) yields MCV. To calculate the MCV in SI units, divide the PCV (L/L) by the RBC count
and multiply by 1000 (a).
13. (Answer: c). Multiplying PCV (%) by 10 and dividing the product by total RBC count (c) yields MCV.
Multiplying Hb by 10 and dividing the product of total RBC count (a) is the formula for determining
MCH. For calculating the MCV in SI units, divide the PCV (L/L) by the RBC count and multiply by
1000 (b). MCHC is calculated by multiplying the Hb concentration by 100 and dividing the product by
the PCV (d).
14. (Answer: b). Mean corpuscular hemoglobin (MCH) is the mean weight of Hb contained within the
average RBC (b), not as mean weight of the RBC compared to Hb within the cell (a). It is recorded in
pictograms (pg), not as femtoliters or g/dL (c and d).
15. (Answer: c). Cats have two kinds of reticulocytes, but only the aggregate form (c), not the punctate
form, should be counted. Reticulocytes are immature erythrocytes, not mature (a) erythrocytes that
still contain ribosomes. Wrights stain causes a polychromatophilic staining, or diffuse, blue-gray
color, not purple-red (b). Because (a) (mature erythrocytes) is not true, (d) is not a viable choice.
16. (Answer: a). Basophilic stippling is the presence of small, blue-staining granules within RBCs (a) often
seen with lead poisoning. This regenerative response does not occur in leukocytes, platelets, or
basophils (b, c, and d) in the presence of lead poisoning.
17. (Answer: c). Osmometry (c) is likely the most representative method of analyzing urine solute osmolar
concentration; it measures the number of dissolved particles in the urine. The urine pH (a) assesses the
bodys acid-base balance, and cystocentesis (b) is a collection method for obtaining samples
suitable for bacterial culture. Finally, whereas urine solute osmolar concentration is considered an
aspect of specific gravity (d), urine SG is not as specific to urine solute osmolar concentration as
osmometry.
18. (Answer: b). Birds and reptiles (b) have NRBCs, which makes determining a WBC count difficult with
the mammalian methods. Ruminants and horses (a, c, and d) are mammals and do not have NRBCs
under normal circumstances.
19. (Answer: c). An increased WBC count is leukocytosis (c). Anisocytosis (variation in size) (a) and
agglutination (clumping) (b) are conditions related to RBCs, not WBCs. Decreased WBC count is
leukopenia (d).
20. (Answer: a). Thrombocythemia (a) is a specific type of thrombocytosis (b) in which a chronic and
extremely elevated platelet count is caused by a bone marrow disorder. Reactive thrombocytosis (c) is
a transient response to trauma, splenectomy, medication, and various diseases not originating in the
bone marrow. Physiological thrombocytosis (d) is commonly caused by movement of platelets from
the storage pools (e.g., spleen) as a result of stress and exercise.

CHAPTER 3 PARASITOLOGY
1. (Answer: c).Toxocara canis eggs are dark brown, thick-walled eggs with a pitted eggshell (c). Clear,
smooth, and thin-walled eggs (a) would be associated with hookworm infection. Yellow-brown thick-
walled eggs with striated shells and asymmetrical plugs (b) describes the diagnostic stage of the
Eucoleus bohmi. Finally, L1 with S-shaped tail lacking a dorsal spine (d) describes the diagnostic
developmental stage of the canine lungworm.
2. (Answer: a).Ancylostoma caninum causes (and is thus contracted by) cutaneous (a) larval migration.
Ancylostoma caninum must mature and molt in the environment before it achieves its infective stage,
so it does not spread by rectal remigration (b), and neither mosquitoes (c) nor fleas (d) spread
Ancylostoma caninum.
3. (Answer: d). Fleas (d) incubate and are the intermediate hosts for Dipylidium caninum. Squirrels (a)
are intermediate hosts of Taenia pisiformis. Ticks (b) do not transmit tapeworms. Cows (c) are
intermediate hosts of Taenia hydatigena.
4. (Answer: a). The prefix oto-is your clue that this is an ear infestation. Head tilt, otitis media
circling, and seizures (a) are all signs of a severe infection of Otodectes cynotis. Head shaking and ear
scratching (b) are signs of a less severe infection. Thickened, wrinkled skin with eventual physical
emaciation (c) is seen in severe (even fatal) infection with a less common ear mite, Psoroptes spp., and
is not usually seen in dogs. Finally, severe tail, leg, or scrotal mange (d) is the sign of infection with
Chorioptes spp., which is also not usually seen in dogs.
5. (Answer: b).Ixodes scapularis is the host to Borrelia burgdorferi (Lyme disease) (b). Rocky Mountain
spotted fever (a), tularemia (c), and Q fever (d) are spread by Dermacentor spp.
6. (Answer: d). Although mainly seen in animals, the flea can be swallowed by humans and thus the
Dipylidium caninum tapeworm (d) can mature in a human host, whereas Cystoisospora canis and
Trichuris vulpis (a and b) are specific to dogs and Parascaris equorum (c) is specific to horses.
7. (Answer: d). Mallophaga (d) are the biting or chewing lice, whereas Anoplura (b) are sucking lice.
Hemiptera (a) are true bugs such as reduviid bugs, and Diptera (c) are the two-winged flies.
8. (Answer: a).Aelurostrongylus abstrusus (feline lungworm) (a) is best diagnosed using the Baermann
test. The others (b, c, and d) are diagnosed by fecal flotation or centrifugal flotation.
9. (Answer: a). One of the great advantages of centrifugal flotation technique is to (a) detect Giardia
oocysts and Trichuris ova more efficiently than with the regular flotation technique. Direct smear is
used to detect protozoa in feces (b), and the number of eggs present per gram of feces (c) requires
quantitative procedures such as the Wisconsin double centrifugation technique and the McMaster
technique. Finally, the standard vial gravitation flotation technique is based on specific gravity of
parasitic material and fecal debris (d).
10. (Answer: c).Oxyuris equi (c) can be lifted off the skin surface using cellophane tape and then taped over
a drop of mineral oil on a slide for visual examination. This is because they are found primarily on the
skin surface and the hair. Thelazia californiensis (a) is found in the conjunctival sac of the eye or in the
tear duct, and Eimeria spp. and Balantidium coli are both intestinal and found in feces (b and d).
11. (Answer: b).Cheyletiella species (b) are referred to as walking dandruff because they resemble
flakes of moving dandruff. Reduviid bugs (a) are referred to as kissing bugs, and Culicoides
spp. (c) are referred to as no-see-ums. Glossina species are referred to as tsetse flies.
12. (Answer: d). The hexacanth is the infective stage of a tapeworm (d). Ticks and mites (a and b) are
visible as adult insects. Pinworms (c) are usually seen in egg form.
13. (Answer: c).Giardia and Leishmania are examples of flagellates (c). Protozoa are one-celled microscopic
parasites that are sometimes classified by movement; in addition to the flagellates, they include ameba
(a), ciliates (b), and sporozoa (d).
14. (Answer: c). The Baermann technique (c) is used to remove lungworm larvae from small amounts of
feces. Centrifugation (a) and McMaster (b) techniques are each used to determine the number of eggs
present per gram of feces; the direct smear technique (d) is used to detect protozoa in feces and
quickly estimate their number.
15. (Answer: c). Tapeworms are cestodes (c). Like flukes (trematodes) (d), they fall into the flatworm
subcategory of the helminths. Nematodes (b) are roundworms, and thorny-headed acanthocephalins
(a) are thought to be intermediate between cestodes and nematodes.
16. (Answer: b). The Knotts test differentiates the blood parasite Dirofilaria immitis from the
nonpathogenic filarial larvae Acanthocheilonema (b). The test is specific to these two and not to the
others (a, c, and d).
17. (Answer: d). Fleas are found on the hosts skin during the adult stage (d) of the life cycle. The
larvae (b) and pupae (a) are found in the bedding and living area of the host.
18. (Answer: a). The prepatent period for Trichuris vulpis is (a) 3 months. Hookworms prepatent
period is 2 weeks (b), and that for Filaroides spp. is 5 to 10 weeks (c). Spirocerca lupi have a prepatent
period of 5 to 6 months (d).
19. (Answer: c). In sucking lice, the head is narrower than the thorax (c). The rabbit fur mite, Listrophorus
gibbus, has a body that is laterally compressed, with a broad head (a). Scutum ornate and basis
capitulum (b and d) both describe the hard tick.
20. (Answer: a).Hyostrongylus rubidus is the red stomach worm of pigs (a). Physaloptera spp. is a stomach
worm of the cat or dog (b and c), and Gasterophilis spp. is a stomach parasite of the horse (d).

CHAPTER 4 CLINICAL CHEMISTRY


1. (Answer: c). Serum is the fluid portion of blood with fibrinogen (c) removed. Albumin (a) is the
largest protein fraction found in both serum and plasma. Globulin (b) is the second largest protein
fraction found in both serum and plasma. FDP (d) stands for fibrinogen degradation products, which
are formed when a clot is broken down and has nothing to do with the initial clot formation.
2. (Answer: d). Lithium heparin (d) would be the anticoagulant of choice because it does not affect the
other analytes being checked. EDTA (a) should not be used because it binds calcium and can affect
this measurement as well as that of some other analytes. Sodium fluoride (b) is used to preserve
glucose in plasma samples, but would falsely elevate sodium levels in the sample, and sodium citrate
(c) is used to run coagulation tests and would falsely elevate the sodium levels in the sample.
3. (Answer: d). Glomeruli function (d) determines the amount of creatinine removed by the kidneys. A
decrease in function would increase the levels measured in the serum or plasma. Creatinine is not
affected by protein levels in the diet (a), the presence of lipemia (b), or the presence of hemolysis (c).
4. (Answer: a). Bleeding into the intestinal tract (a) provides additional protein for digestion, thereby
increasing the protein level in the diet, which can cause an increase in serum urea nitrogen. Excessive
muscle activity (b) does not affect serum urea nitrogen, but can affect the CPK. Cushings disease
(c) causes an increase in cortisol but does not affect serum urea nitrogen. Inadequate fasting (d) can
cause an increase in lipemia and potentially glucose levels, but not serum urea nitrogen.
5. (Answer: a). The renal threshold is (a) the level above which the kidney is no longer able to
effectively remove the glucose from the renal filtrate, causing loss or spillover into the urine. There is
no (b) maximum level of measurable substance, although dilutions may be necessary to determine the
actual value. The minimum level of a substance in urine (c) is usually zero. If a substance is not found
in the urine (d), it has definitely not reached the renal threshold.
6. (Answer: c). Because RBCs continue to metabolize and use glucose even after removal from the body,
they should be (c) separated from the serum as soon as possible; otherwise, an artificial decrease in
the glucose level can be caused by prolonged contact of the serum or plasma with RBCs. A fasting
sample is preferred to one collected right after an animal has eaten (a). Chemistry samples should not
be collected using EDTA (b). Because it is not necessary to wait for the sample to clot, or wait for that
clot to retract so that the serum can be collected, a plasma sample would actually be quicker and
easier to collect.
7. (Answer: c). Pancreatic lipase immunoreactivity (PLI) (c) is the test of choice because serum PLI
concentrations are produced almost exclusively from the exocrine pancreas. Serum lipase (a) is
produced by organs other than the pancreas and measures the digestive level, not the function, of the
pancreas. Trypsinlike immunoreactivity (b) is considered the test of choice for detection of exocrine
pancreatic insufficiency (EPI) in dogs but not exocrine function. Bile acids (d) measure liver, not
pancreatic, function.
8. (Answer: d). Amylase breaks down complex carbohydrates such as starches (d) and glycogen. Lipase
is used to break down fatty acids (a), and glycerol (b) is produced by the breakdown of long-chain
fatty acids. Amino acids (c) are produced by the breakdown of proteins.
9. (Answer: d). All three statements (d) are correct: the unconjugated or indirect bilirubin is lipid-
soluble and bound to proteins (a), but only direct bilirubin is water-soluble (b). Indirect bilirubin is
calculated by subtracting direct bilirubin from the total bilirubin (c).
10. (Answer: a). The albumin-to-globulin ratio (A/G) is calculated by dividing the albumin results by the
globulin results (a). In dogs, horses, and sheep the A/G is > 1; in cattle, pigs, and cats the A/G ratio
is 1 (b). A/G should be evaluated along with the protein profile, rather than independently (c).
Thus it is not possible that all three are correct (d).
11. (Answer: b). Electrophoresis (b) is the passing of an electric current through a sample, separating it
into different fractions. Globulin fractions, along with albumin, can be separated by electrophoresis.
Clotting (a) involves the protein fibrin, not globulin, and globulin is a measured value not a calculated
value (c). A refractometer (d) measures total protein not separate proteins.
12. (Answer: b). ALT (b) is produced primarily by the liver in dogs and cats. AST (a) can be produced by
tissues other than liver, and is therefore not liver specific, especially in dogs and cats. AP (c) can be
produced by tissues other than the liver and is therefore not liver specific, especially in growing
animals. CPK (d) is not produced by the liver but rather by muscle activity.
13. (Answer: a). It is not true that most of these tests can be done at room temperature (a). Instead, most
enzymatic tests are run at 86 F (30 C), using nonhemolyzed, nonlipemic samples (b) in which the
sample is separated from the cells (c); otherwise, glucose and some electrolytes will be affected if a
sample is not separated. The instructions must be followed for accurate results (d).
14. (Answer: a). Increases in albumin are rare (a); they are sometimes seen in shock. Increases in alpha-
globulins (b) may be seen with inflammation and infections, antigenic stimulation, and neoplasia or
abnormal immunoglobulin production. Increases in beta-globulins (c) may be seen with inflammation
and infections, antigenic stimulation, neoplasia, or abnormal immunoglobulin production. Increases in
gamma-globulins (d) may be seen with inflammation and infections, antigenic stimulation, neoplasia,
or abnormal immunoglobulin production.
15. (Answer: d). All four answers (d) are correct. If the liver is recycling bile acids normally (a), only a
small amount will be found in the bloodstream. Bile acids do increase before bilirubin increases are
seen (b); therefore this is a more sensitive test for evaluating liver function. Bile acids are released from
the gall bladder, travel down the small intestines, and are filtered from the blood by the liver (c).
16. (Answer: b). Phosphate, phosphorus, magnesium and potassium (b) are intracellular, whereas the
others, such as calcium (a and c), sodium (c), bicarbonate (d), and chloride (c and d) are not.
17. (Answer: a). Bicarbonate (HCO ) (a) does carry an electrical charge and is technically an electrolyte,
3

but it is involved in acid-base balance and is therefore not evaluated with the other electrolytes.
Phosphorus (b) is one of the major intracellular electrolytes, and sodium (d) is one of the major
extracellular electrolytes. Albumin (c) is a plasma protein not an electrolyte.
18. (Answer: b). Sodium and chloride levels have a close relationship (b), and each plays a major role in
the distribution of water and the maintenance of osmotic pressure of fluids in the body. Calcium and
phosphorus (a) are involved primarily in bone formation and maintenance, and magnesium and
potassium (c) are intracellular electrolytes and have little effect on fluid balance in the extracellular
space. Calcium works predominately with phosphorus, not chloride (d).
19. (Answer: c). RIM testing (c), which stands for rapid immunomigration testing, has also been called lateral
flow immunoassay, or immunochromatography. TLI (a) refers to trypsinlike immunoreactivity assay;
ELISA testing (b), or enzyme-linked immunosorbent assay, is an accurate way to detect specific
antigens or antibodies such as viruses, bacteria, parasites, or hormones in serum and plasma. Latex
agglutination testing (d) uses small, spherical latex particles coated with antigen suspended in water;
with positive samples, formation of an antibody-antigen complex occurs and is seen as agglutination.
20. (Answer: a). Triglycerides (a) are the fat fraction most responsible for lipemic blood samples, often as a
result of diet, metabolic disease, or inadequate fasting. Lipase and amylase are enzymes (b and c), not
fats; and cholesterol (d) is a clear fat, whereas lipemic serum is cloudy.

CHAPTER 5 CYTOLOGY
1. (Answer: d). Methanol 95% (d) is the preferred fixative for most cytology preparations. Tissue
samples for histopathological examination are usually fixed in 10% neutral buffered formalin (a).
Phosphate (b) is not used as a fixative, and EDTA (c) in saline is used with marrow smears.
2. (Answer: b). The fatty material in chylous effusions (b) interferes with light passage through the
refractometer, whereas color of samples (a) may vary and samples from a hemorrhagic area (c) will
not have a great effect on assessing total protein with a refractometer. If a sample is characterized as a
transudate (d), this does not affect the refractometers ability to accurately weigh total solids.
3. (Answer: c). Rayon swabs (c) are preferred over cotton because cotton, whether on a swab, a cotton
ball, or an applicator stick (a, b, and d), can inhibit bacterial growth and oxygen can become trapped
in the fibers, inhibiting the recovery of anaerobic bacteria.
4. (Answer: c). The patient is usually standing or in ventral (c) recumbency for removal of pleural fluid.
Dorsal or lateral recumbency (a and b) is better for cystocentesis, and lateral or sternal recumbency (b
and d) is used for removal of pericardial fluid.
5. (Answer: a). The modified compression smear (a) subjects cells to lower shear forces than the
standard compression (c) preparation. A starfish smear (b) is most useful for highly viscous samples,
and a wedge smear (d) is used with aspirate smears to produce a film similar to that used for a whole
blood differential count.
6. (Answer: b). The starfish smear (b) allows spreading of cells in a viscous sample that would not
spread with a standard compression (c) or modified compression (a) preparation. A wedge smear (d)
is used with aspirate smears to produce a film similar to that used for a whole blood differential count
7. (Answer: c). Cytology samples require increased fixative times in contrast to blood films. The
prepared cytology slides should remain in the fixative for at least 2 to 5 minutes (c). A range of 30 to
60 seconds (a and b) is not long enough, and 10 minutes (d) is not the required minimum; longer
fixative times will improve the quality of the staining procedure and will not harm the samples.
8. (Answer: b). New methylene blue (b) will stain nuclei, mast cell granules, and most infectious agents.
Gram stains (d) work better with gram-negative and gram-positive organisms, which each stain a
different color. Mast cell granules may not stain with Romanowsky stains such as Giemsa (a) or Diff-
Quik (c).
9. (Answer: b). The term karyorrhexis (b) describes a fragmented nucleus. Pyknosis (a) describes a
condensed nucleus; karyolysis (c) refers to a loss of nuclear membrane, and hypersegmentation (d)
refers to a nucleus with more than five lobes.
10. (Answer: d). Mesothelial cells (d) are characterized by a fringe border, or corona, and finely reticulated
nuclear chromatin. Mast cells (a) are round to oval, and macrophages (b) are oval to pleomorphic with
lacy to condensed chromatin. Plasma cells (c) are oval with an eccentric nucleus, a basophilic
cytoplasm, and a perinuclear clear zone.
11. (Answer: d). Granulomatous inflammation (d) is characterized by the presence of more than 70%
mononuclear cells and few neutrophils, a pyogranulomatous inflammation (a) consists of
macrophages and 50% to 75% neutrophils, and a purulent inflammation (b) has more than 85%
neutrophils. An eosinophilic inflammation (c) consists of greater than 10% eosinophils, but few mast
cells, plasma cells, and lymphocytes.
12. (Answer: c). Hyperplasia (c), or benign neoplasia, is characterized by homogeneous populations of the
same cell type with no evidence of malignant characteristics within cells. The other
threeaniscocytosis, macrocytosis, and pleomorphism (a, b, and d) are all criteria for malignancy.
13. (Answer: b). A hematoma (b) is a noninflammatory and non-neoplastic lesion, whereas histiocytoma,
TVT, and mesenchymal (a, c, and d) are all types of neoplastic tumors.
14. (Answer: a). Malassezia (a) is a common fungus recovered from ear cytology samples. Otodectes (b) is a
parasite found in the ear. Simonsiella (c) is normal bacterial mouth flora, and Blastomyces (d) is a fungal
inflammation found in the lungs.
15. (Answer: c).Simonsiella (c) is normal flora of the oral cavity. Malassezia (a) is a common fungus
recovered from ear cytology samples. Otodectes (b) is a parasite found in the ear. Blastomyces (d) is a
fungal inflammation found in the lungs.
16. (Answer: a). A variety of cell types are present in a normal (a) lymph node, including plasma cells,
macrophages, and small, medium, and large lymphocytes, but with a majority (75% to 90%) of small
lymphocytes. Neoplastic findings (b) would show lymph node cells with abnormal nuclear features or
presence of a large number of blast cells. Inflammation (c) would show an increase in neutrophils,
eosinophils, and macrophages. Finally, a metastatic sample (d) would show neoplastic cells from other
body tissues that spread to lymph nodes.
17. (Answer: a). Hematoxylin and eosin stain (a) is preferred for core biopsy samples. New methylene
blue (b) will stain nuclei, mast cell granules, and most infectious agents. Papanicolaou stain (c) is
commonly used in human gynecological examinations. Giemsa stain (d) provides satisfactory staining
of cytological specimens.
18. (Answer: a). Increased numbers of mature lymphocytes (a) is a characteristic of chylous effusion.
Lymphoblasts (b) are immature lymphocytes; and macrophages (c) are large phagocytic WBCs in
blood, lymph, and connective tissue. Polychromatophilic rubricytes are a type of nucleated RBC (d).
19. (Answer: b). A variety of cell types are present in vaginal cytology samples collected during proestrus
(b), including RBCs and parabasal, intermediate, and superficial epithelial cells. An estrus sample (a)
contains all superficial cells, with many appearing to be anuclear or with small pyknotic nuclei. RBCs
may be present, but no neutrophils. A metestrus (c) sample shows parabasal and intermediate cells
replacing superficial cells. Neutrophils increase and RBCs are usually absent. An anestrus sample (d)
is made up predominantly of noncornified squamous epithelial cells and some neutrophils, but no
RBCs.
20. (Answer: b). The nonaspiration method is the preferred sample collection method for vascular
samples (b) such as the liver and spleen. A type of fine-needle procedure, it is also referred to as the
capillary technique or stab technique because it uses a needle or a syringe without the plunger
attached to a needle. That means that it is not as useful for solid, fatty, or fibrinous sample collection
(a, c, and d).

CHAPTER 6 DIAGNOSTIC MICROBIOLOGY AND MYCOLOGY


1. (Answer: b). Moisture forms during the incubation process and can smear the forming colonies, so
media plates are incubated upside down to prevent condensation from dripping onto cultures (b).
Light and gravitys effects on colony formation (a and d) are not issues with bacterial cultures,
and the technician is exposed when examining the colonies (c).
2. (Answer: d). All of these are good reasons to use an outside laboratory (d), and the vast number (a),
the lack of capacity for clinically trained staff (b), and the challenges of difficult-to-grow
microorganisms (c) are the main ones.
3. (Answer: c). By inhibiting the growth of certain bacteria (c), these media allow others to grow.
Allowing no bacterial growth (a) would be counter-productive, differential media is a different type
of agar (b), and no media change color immediately (d).
4. (Answer: c). Sterile means no bacteria or fungi are normally found in a body fluid or cavity (c). This
means that it does not contain any bacteria or fungi, including normal flora (a, b, and d).
5. (Answer: d). Both (d) sterile catheterization (b) and cystocentesis (c) are acceptable for a culture. Free
catch urine (a) may contain normal flora from the skin and genital area.
6. (Answer: c). Blood should be sterile, and low numbers of bacteria in septicemia require that a liquid
medium (c) be used to grow the organisms first, instead of inoculating immediately (a) or using
EDTA blood only (b). A culture is normally selective or specific to certain organisms, rather than
many at once, as in (d).
7. (Answer: b). Because of the high numbers of bacteria present (contrary to choice [a]) in feces, special
media are required (b). Gram stains (c) can help identify some pathogens immediately, and the
patient species should always be taken into account for species-specific pathogens (d).
8. (Answer: d). Both b and c are correct (d). Special media are required for fungal cultures, so they
cannot use the same types of media as bacterial cultures (a), and they are incubated for a longer time
(c) and protected from light while incubating (b).
9. (Answer: d). It is best to refrigerate shipping samples to help them stay viable (d). Samples should
not be held (a) but should be shipped as soon as possible. A padded envelope by itself (b) is not
sufficient for shipping. Culturettes (c) have a small amount of liquid or gel at the swab end that helps
the organisms survive transport.
10. (Answer: a). The catalase test differentiates between staphylococci (which are catalase positive) and
streptococci (which are catalase negative) (a). The coagulase test is used to differentiate among
staphylococci (b). Hemolysis is determined from the blood agar plate (c). Yeasts are catalase positive
(d).
11. (Answer: d). The Gram stain is used to differentiate gram-positive and gram-negative bacteria (d). It is
not only important (a) but is one of the most basic tests in microbiology. It is not just used for gram-
negative bacteria (c) but is also used to differentiate gram-positive and gram-negative bacteria (d). An
acid-fast test is used to determine whether bacteria are acid-fast (b).
12. (Answer: a). Gram-positive bacteria will grow not only on the TSA plate (b) (which grows most
organisms) but also on the CNA plate (which inhibits gram-negative growth) (a). They will also grow
in broth (d). MAC agar growth indicates gram-negative bacteria (c).
13. (Answer: c). Beta-hemolytic streptococci will show complete hemolysis of the blood in the TSA plate,
marked by clear agar surrounding the colony (c). A greenish zone of hemolysis (b) is called alpha
hemolysis. No hemolysis (a) is gamma hemolysis. Special agar is needed to determine if a Streptococcus
sp. is bile-esculin positive (d).
14. (Answer: d).Neisseria and Moraxella spp. (a) are rare (c) in a typical veterinary clinic, making (d) the
correct answer. Only gram-negative rods can be identified on the API 20E strip (b).
15. (Answer: b). MAC agar differentiates lactose-fermentation by turning those colonies pink or purple
(b). Nonlactose fermenters are clear (a). In a Gram stain, gram-positive bacteria stain purple or dark
blue (c). Hemolysis is determined on a TSA plate (d).
16. (Answer: d). All these characteristics help identify these two organisms (d). They can be identified by
the API20E system (c), even though they are not typical enteric bacteria. They are gram-negative
coccobacilli (a) and usually only grow on the TSA plate (b).
17. (Answer: a). Dermatophytes are the fungi that cause ringworm (a). They require special media to
grow (0.9% saline is a diluent only) (b). They are often encountered in the veterinary laboratory (c) and
require microscopic examination for identification (d).
18. (Answer: d). Prolonged antibiotic use (contrary to choice [a]) and a bacterial infection (d) can lead to a
yeast infection. Candida do not typically cause infections in healthy animals (c); they are part of the
normal flora (b) and can be found in mucous membranes, especially in the gastrointestinal tract
(including the mouth), genital tract, respiratory tract, and ears. They must be present to cause an
infection.
19. (Answer: c). Ears, throat, and the environment (c) can have fungi as part of their normal flora. Blood,
urine, and spinal fluid (a and b) should normally be sterile. Sterile abscesses (d) grow no
microorganism at all.
20. (Answer: c). Aspergillus can cause disease in immune-compromised individuals often through
ingestion affected feed and hay (c). Though found in the environment (a), this does not hamper their
ability to cause disease. Contrary to choice (b), they are easily grown in the veterinary laboratory. They
are saprophytic fungi, not dimorphic fungi (d), which exhibit both yeast and hyphal forms.

CHAPTER 7 RADIOGRAPHY
1. (Answer: b). As a wavelength increases, the frequency decreases (b), not the reverse (a). Frequency
and wavelength are inversely proportional, so a change in one does affect the other (making c and d
untrue).
2. (Answer: a). Ionizing electromagnetic radiation is characterized by the energy contained in a photon
(a). Ionization is a type of energy transfer induced by causing x-rays to interact with matter (b);
inappropriate exposure can result in tissue damage (c). An x-ray is a penetrating, ionizing
electromagnetic radiation that has a wavelength much shorter than that of visible light, not longer (d).
3. (Answer: c). X-rays form a heterogeneous beam, not a homogeneous beam (c) that travels in a
straight line, not a wavy one (c). They are electrically neutral and have no mass (d) and can penetrate
living tissue (a), causing some substances to fluoresce (b).
4. (Answer: d). Bremsstrahlung and braking radiation are the same thing (d) and make up most x-rays
in the diagnostic range. Bremsstrahlung (or braking) radiation (a and b) occurs when an incoming
projectile electron moves very close to the nucleus, losing energy as it passes. Characteristic radiation
(c) occurs when the energy of a projectile electron interacts with a K-shell electron and removes it
from its orbit.
5. (Answer: b). The kilovoltage (kV) controls contrast (b). At lower kV, the photoelectric effect results in
total absorption of the energy of the incoming x-ray photon, meaning both the absorbed dose and the
contrast increase at lower kV. The quantity of radiation in milliamperage per second is represented by
the abbreviation mAs (a). SID (c) or source-image distance affects magnification and detail. RAD (d)
stands for radiation absorbed dose and measures the energy transferred by radiation to a body part.
6. (Answer: a). When film density is too light and the technician suspects that the problem is related to
exposure time (mAs), doubling the density from 10 mAs to 20 mAs means a change to 200 mA per
0.10 second or 20 mAs (a) while (b) does not double the mAs. If you increase both kV and mAs (c and
d) you are more than doubling the density plus you are decreasing the contrast with the change in kV.
7. (Answer: a). An increase in SID and a decrease in SOD (a) help minimize magnification and
penumbra. An increase in the source-object distance (b and c) would cause more magnification and
penumbra, and likewise, a decrease in SID (c and d) increases magnification.
8. (Answer: d). The heel effect is most noticeable when using largest film size (a and b) and a short
source-image distance (SID) (a and c). Thus a smaller film and longer SID (d) will help make the heel
effect less noticeable.
9. (Answer: c). Increasing film speed increases the darkness of the image (c), not the other way around
(b). Use of a grid not compensated for would make the film lighter (a), and improper stirring of
solutions causes reticulation (d) (a network of wrinkles resulting from sharp temperature differences
in the solutions).
10. (Answer: c). A functional safelight will process to a density of 0.50 or very light silver gray (c), as
opposed to streaked, totally dark, or clear (a, b, and d).
11. (Answer: a). During development, agitation (a) during the developer fixer process ensures that fresh
chemistry is moved onto the film and used chemicals are moved away from the film. Developer (c)
primarily functions to reduce, or convert, the exposed silver halide crystals of the film emulsion to
black metallic silver. The fixers primary functions (b) are to remove and clear away the
unexposed, undeveloped silver halide crystals and harden the film to make it a permanent record.
Although proper time-temperature is important (d), only by agitating the film can fresh chemicals be
moved over the film and used chemicals be moved away from it.
12. (Answer: d). The phosphors used today primarily emit in the yellow-green light part of the spectrum
(d) not in the blue-green part of the spectrum (a). There is greater x-raytolight conversion with
current phosphors, not minimal x-raytolight conversion (b), resulting in decreased exposure
factors (not increased exposure factors [c]) required to produce the same density on a film.
13. (Answer: d). The lateral along with the craniocaudal view (d) are the best views of the canine elbow.
The patient is lying in ventral recumbency so the term for the elbow is the craniocaudal view. In this
position it would be called dorsopalmar view (a) for the carpus, metacarpus, and digits, while the
plantarodorsal (b) is the term for the metatarsus and corresponding digits. The caudocranial (c) is the
term, used for the shoulder, scapula and humerus, when the patient is lying in dorsal recumbency.
14. (Answer: a). In an accurate view of the elbow, the cranial border should be the distal third of the
humerus, with the caudal border at the proximal third of the radius or ulna (a). The junction of the
femur and tibia or fibula (b) would indicate the hind leg rather than the forelimb. Including two thirds
of the bones distal and proximal (c) could adversely affect the resultant image, and borders consisting
of the carpus and ulna or radius (d) would focus on the wrong joint entirely.
15. (Answer: c). The big three tenets of radiation safety include minimum time settings for the
examination, combined with keeping as far as possible from the x-ray beam, and always using
shielding (c). Conversely, you would increase your risk for harmful effects if you consistently used
maximum time settings (a and b), stayed as close as possible to the patient and beam (a), and neglected
constant use of protective apparel (d).
16. (Answer: b). Contrast is increased at a lower kV (d) and with the use of a grid (a), with the collimator
aperture set to narrow the field (c). Increasing time and chemical temperature (b) will darken the film
and decrease the contrast.
17. (Answer: a). The developer primarily functions to reduce, or convert, the exposed silver halide crystals
of the film emulsion to black metallic silver (a). It is the fixer that works to (b) remove and clear away
the unexposed, undeveloped silver halide crystals and harden the film to make it a permanent record.
Calcium tungstate (c) is a phosphor rarely used since 1981, when it was no longer recommended. A
latent image (d) is not produced by the developer, but by accidental ionizing radiation exposure, light,
heat, or fumes before the film has been developed.
18. (Answer: d). Only CR converts photons to metastable electrons (d). Both conventional and computed
radiography use a scintillating screen (enclosed within a cassette) (b) that emits light (a) and must be
processed in a separate unit (c).
19. (Answer: b). This study as described is incomplete because the technician fails to provide an exposure
at the 15-minute interval and hourly after 90 minutes if requested by the veterinarian (b). It should
also include an immediate exposure at 0 and 15 minutes for a complete study including the stomach
(d) and a 90-minute exposure (c) to be complete (a).
20. (Answer: a). A histogram (a) in digital radiography takes into account the frequency of appearance of
a given object characteristic. For example, extremities are imaged as high-contrast structures (little soft
tissue and much dense bone), whereas abdomens are imaged as low-contrast subjects with much soft
tissue and very little dense bone. The algorithm (b) is the formula whereby the image is produced
from the raw data of the electrical signal. Typically, this is determined within the manufacture of the
digital plate (c) and the computer interface (d).

CHAPTER 8 ALTERNATIVE IMAGING TECHNOLOGY


1. (Answer: a). The definition of hyperechoic is when something is visually lighter or brighter than the
tissue around it (a). If something is darker than the surrounding tissue (b), it is hypoechoic, and if it is
the same as the surrounding tissue (d) it is isoechoic. If no sound wave penetrates and a black shadow
appears (c), this may be acoustic shadowing.
2. (Answer: c). A 7.5-MHz transducer gives increased detail. It is best used on small dogs and cats (not
d) It does not penetrate as deeply (a) as the 5-MHz transducer and signal attenuation is greater, not
decreased (b) but it provides better detail. Larger dogs are best scanned with a 5-MHz transducer
3. (Answer: b). A Doppler shift is a change in the frequency (b) as a result of motion, as opposed to
acoustic impedance (d). Volume of either a perceived sound (a) or a sound source (c) does not affect a
Doppler shift, although, a received frequency in Doppler imaging is either higher or lower than that
transmitted by the source, and information comes not only from the Doppler shift but also by sound
audible to the ear or analysis with an instrument (again, neither of which defines a Doppler shift).
4. (Answer: a). When an ultrasound beam interacts with a large, curvilinear, highly reflective surface, it
can create a mirror image (a) artifact of the organ itself. It can be tempting to choose refraction (b),
which is a wedge-shaped area that can appear distal to a curved structure where no sound beam is
present. Reverberation (c) occurs when the ultrasound beam is reflecting back from a gas interface
and then bounces back and forth between the probe and the gas, resulting in evenly spaced parallel
white lines. A slice thickness artifact (d) occurs when part of the ultrasound beams width is
outside a cystic structure, resulting in an image that produces echoes that mimic sediment.
5. (Answer: b). The transverse plane will create an image representing the ventrodosal (b) radiograph.
In creating an ultrasound scan of the abdomen, each organ should be imaged in a minimum of two
planessagittal and transverse. These two planes mimic the appearance of radiographs, sagittal (a)
representing a lateral (c) radiograph (patients cranial side to the left) and transverse representing
a ventrodorsal radiograph (patients right side to the left). Neither plane represents a medial (d)
radiograph.
6. (Answer: c). Normal adrenal glands are uniformly hypoechoic (c) to the surrounding fat on
ultrasound examinations, which means that they are darker than the surrounding tissue. Conversely,
if they were lighter than the surrounding tissue, they would be described as hyperechoic (a), and if
they showed the same echogenicity as the surrounding tissue, they would be termed isoechoic (b).
Anechoic (d) means that the image is completely black, as would be the case with a fluid.
7. (Answer: d). For a sagittal image, the probe is oriented from cranial to caudal (d), not caudal to
cranial (c). In transverse, the probe is oriented from right to left (a) and not left to right (b).
8. (Answer: a). The pancreas (a) is not only difficult to see as a distinct structure in ultrasound in
contrast to the bladder, liver, or kidney (b, c, and d), but it is also most difficult to correlate
sonographic appearance with presence or absence of disease in the pancreas
9. (Answer: b). In a deep-chested dog, sonography of the liver requires cranial angulation of the probe
(b). The correct left and right margins are the spleen or stomach (left) and right kidney (right
margin)not the appendix and spleen (a). The liver also should be imaged through the 10th to 12th
intercostal spaces, not between the 8th and 10th intercostal spaces (c), and in deep-chested dogs, the
transducer needs to be moved a great distance along the costal arch, not pivoted or fanned (d) as in
small dogs.
10. (Answer: c). The wall of intestinal tract consists of five layers in this order: mucosal surface, mucosa,
submucosa, muscularis, serosa (c). The other orders (a, b, and d) are incorrect.
11. (Answer: a). Because of the shortage of hydrogen in bones, CT is better than MRI for viewing bony
lesions (a). MRI, however, is preferred for pericardial effusion, brain disorders, and large vessel
thrombi (b, c, and d).
12. (Answer: b). With ultrasound, a solid lesion contains many echoes, which can be spread throughout
(b), as opposed to having no internal echoes (d), which is the case with a cystic lesion. It shows no
distance enhancement (a), which is also typical of a cystic lesion. Irregular borders (c) best describes
mixed lesions.
13. (Answer: c). As the depth of a structure increases, the returning echoes become weaker; TGC sliders
increase the gain at different depths of the organ (c). It is the gain button that controls the overall
brightness of the image (a), and focus is a method used to both move the focal point closer to the
image (b) and to narrow the width and improve resolution (d).
14. (Answer: b). In through-transmission (b), and far-field enhancement, there is minimal attenuation of
the beam as it passes through a fluid-filled structure; the beam on the far side is more intense than the
beam at the same depth that misses the fluid cavity, resulting in echogenicity distal to the fluid-filled
structure. Acoustic shadowing (a) occurs when the majority of the ultrasound beam is reflected or
absorbed at the interface, and reverberation (c) occurs when the ultrasound beam is reflecting back
from gas and then bounces back and forth between the probe and the gas. Refraction (d) occurs along
curved spaces with variable acoustic impedances, producing a wedge-shaped area distal to the
structure where no sound beam is present.
15. (Answer: a). Each organ should be imaged in a minimum of two planessagittal and transverse (a).
These two planes mimic the appearance of radiographs, sagittal representing a lateral radiograph
(patients cranial side to the left) and transverse representing a ventrodorsal radiograph
(patients right side to the left).
16. (Answer: c). Cat adrenals are oval (c), not tubular, oblong, or peanut-shaped (a, b, and d), They are
approximately 4 mm in size (c).
17. (Answer: d). It is the colon (d) that can be distinguished by its consistent location and striking acoustic
shadow and reverberation artifact. Reverberation occurs when the ultrasound beam reflects back from
the gas and bounces back and forth between the probe and the gas. The other organsthe heart,
prostate, and uterus (a, b, and c)are not nearly as definitively consistently associated with a specific
artifact.
18. (Answer: d). Sound requires a physical medium through which to travel, such as a solid (a), liquid (b
and c), or viscous matter. The ultrasound waves are unable to pass through air and bone (d) because of
the large difference in acoustic impedance with other soft tissues, causing almost all the sound to be
reflected back to the transducer, creating a barrier to sound transmission.
19. (Answer: d). The actual loss of intensity is termed attenuation. Absorption (a), the production of heat
as sound passes through soft tissue or scatter (b), the reflection of sound both cause attenuation. The
term amplitude (c) describes a display modethat is, how the returning echo or image appears on
the screen.
20. (Answer: d). The nuclei of radionuclides (d), also known as radioactive isotopes or radioisotopes,
decay as a result of an imbalance of neutrons (c) that causes the instability. An isotope (a) is an atom
that has the same atomic number but a different atomic massthat is, the same number of protons
(b) but different number of neutrons (c).

CHAPTER 9 IMMUNOLOGY AND VIROLOGY


1. (Answer: a). During primary immune response, B cells are stimulated to secrete IgM antibody, which
is largely confined to the vascular system (a). Most plasma cells produce IgG antibody molecules
during secondary and subsequent immune responses, not during the primary response (b), and can
cross the placental barrier in some species, but not as a primary response (c). Finally, IgD
antibodys primary role is as an antigen receptor for B cells, and the antibody is found on
lymphocyte membranes in negligible amounts (d).
2. (Answer: c). Both innate and adaptive immunity include cell-mediated phagocytosis (c), which is the
bodys first line of defense for containing and halting the spread of the pathogen. Macrophages
and T-helper cells produce specific cytokines, or lymphokines, and not the other way around (a).
Lymphokines such as interleukin-4 (IL-4) cause B-lymphocytes to clone and produce memory cells
and antibodies, not vice versa (b). Finally, specific lymphokines such as IL-4 and IL-6 cause B-
lymphocytes to clone and produce memory cells and antibodies; they are not stimulated by them (d).
3. (Answer: b). Examples of lymphokines are interleukin (IL) IL-1, IL-2, IL-4, and IL-6, which stimulate
the production of T-helper cells, memory cells, antibodies, and more interleukins (b). NK cells, TNF,
and cytotoxic T cells (a, c, and d) are not lymphokines.
4. (Answer: d). IgA is found in body secretions (d), including tears, mucus, and colostrum. IgE (b) is
found in minute levels in the plasma of healthy animals; IgG (a) is produced in plasma cells and can
cross the placental barrier in some species. IgM (c) is secreted by B cells and is confined to the vascular
system.
5. (Answer: c). IgE (c) boosts inflammatory reactions and an excessive localized reaction indicates an
allergy. IgM, IgA, and IgD (a, b, and d) each offer immune responses but are not associated with
allergic reactions.
6. (Answer: a). In passive immunity, antibodies are donated (a), in that the individuals
immune system was not stimulated to produce the antibodies (nor any memory cells). One example is
immunity that reaches the fetus through the placenta (d) or is passed to a newborn through
colostrum. It is short-term, rather than long-term (b) and can be artificially acquired (c).
7. (Answer: b). In seroconversion or active immunity (b) the individuals own immune system
produced the antibodies, and therefore long-term immunity occurs. Acquired immunity (a) refers to
those that occur after birth; artificial immunity is medially induced; and passive immunity (d) is
acquired temporarily from other individuals.
8. (Answer: a). It is extremely important to follow protocol regarding fetuses and newborns, especially
in the timing (a) of administration. If vaccines are given too soon, the maternal antibodies may block
the neonates immune system from responding, so no immunity occurs. The other statements
regarding SC or IM route (b) and the possibility of adverse reactions (c) and inability to guarantee any
vaccine (d) are all true.
9. (Answer: d). In antigenic drift, the correct choice is (d) because both (a) and (b) are truethe virus
combats an immune response (a) by mutating its genome and epitopes (b) so that previous antibodies
are unable to bind.
10. (Answer: c). There is no evidence that polyphagia (c) occurs. Common clinical reactions can include
urticaria (a), salivation (b), dyspnea (d), and vomiting and incoordination.
11. (Answer: d). Viruses are classified on the basis of all but the (d) organ affected. They are classified in
several ways: by morphology (a), or shape as seen on electron microscopy; the genetic composition (b)
of their nucleic acid core, or genome; and whether the virus possesses an envelope (c). Because of their
pathological distribution, viruses are not classified on the basis of the organ affected (d).
12. (Answer: a). Naked viruses are less susceptible to destruction, not more susceptible (a), primarily
because they lack an envelope, which makes them more tolerant of colder conditions (d). Enveloped
viruses are easily killed with bleaching or freezing and thawing. They are more refractory (c), and
steam sterilization (b) is recommended to kill all viruses at the temperature of 250 F (121 C) at 15
lb per square inch (psi) for 30 minutes.
13. (Answer: c). Some double-stranded DNA viruses can incorporate their DNA sequences into host cell
DNA and be replicated during mitosis (c). The other terms are related to the four basic stages of
replication seen in most viruses: attachment (a), penetration-uncoating (b), replication, and assembly-
release (d).
14. (Answer: b). Viral infections can predispose a patient to secondary opportunistic pathogens (b). Often,
clinical signs are silent, nonapparent, or subclinical, as opposed to always manifesting rapidly (a). Two
things that make viral infections so insidious are that persistent carriers are not easily identified
because their lack of immediate overt signs may result in a transient carrier state (c). Finally, oncogenic
viruses have an RNA nucleic core; they are potentially cancer-causing agents (d).
15. (Answer: a). Specimens for viral testing should be kept cool, 34 to 39 F (1 to 4 C) (a) and no
warmer (b), but do not freeze (c). They definitely should not be kept at body temperature (d).
16. (Answer: c). Enzyme-linked immunosorbent assay (ELISA) can screen for diseases such as feline
leukemia virus (FeLV) (d), parvovirus (a), and heartworm (b), but not rabies (c).
17. (Answer: d). The FA test uses antibodies of known specificity that bind viral antigens, which is
visualized through conjugation (labeled) with a fluorescent dye (d). Accuracy depends on the
application to appropriate specimens that are in good condition (fresh with little to no autolysis) (b). In
ELISA, a known specific antibody is adsorbed, usually in a small plastic well in a well plate or a
specially designed membrane (a). Finally, it is in the latex agglutination (LA) test that antiviral
antibodies are adsorbed to microscopic latex beads (c).
18. (Answer: b). Direct Coombs looks for antibodies bound to the surface of red blood cells (b), not
antigens (c). It does not test or analyze white blood cells (a); only the indirect Coombs test detects
antierythrocyte antibodies in the serum (d).
19. (Answer: d). The radioimmunoassay (RIA) detects the quantity of hormones, drugs, or other antigens
in specific species (d). Precipitation resulting from interaction of antibody and soluble antigen (a)
describes the immunoprecipitation test. Western blotting is used in detection of viruses such as FIV
and FeLV (b). Finally, tests such as rapid immunomigration and ELISA determine an animals
immune reaction to a particular infectious agent (c).
20. (Answer: c). Because of the granular appearance of the latex beads in solution (c), it is very important
to run positive and negative controls simultaneously in the LA test. Reagent is added to the
patients washed red blood cells (a) in the Coombs test, not the LA test. A radioisotope
directly binds with the antigen or antibody in the sample (b) in the radioimmunoassay, not the LA
test. Finally, the sandwich formation (d) is produced in the ELISA test.

CHAPTER 10 SANITATION, STERILIZATION, AND


DISINFECTION
1. (Answer: c). Table 10-1 in the text lists microbial agents from most to least resistant in this order (c)
and also includes prions, bacterial spores, coccidia, mycobacteria and acid-fast bacteria, small-sized
viruses, fungi, vegetative bacteria, and lipid or medium-sized viruses. Spores are more resistant than
fungi, eliminating choice (a); vegetative bacteria are more resistant than lipophilic viruses (as in b);
and hydrophilic are more resistant than lipophilic viruses (d).
2. (Answer: c). Disinfection is the destruction or removal of pathogens from nonliving objects by
physical or chemical methods (c) (see Degrees of Microbial Control). The elimination of all life from
an object (a) is the definition of sterilization. Reduction of microbial populations to levels that are
considered safe (b) is the definition of sanitation. The physical removal of organic and inorganic soils
(d) is the definition of cleaning.
3. (Answer: c). The suffix static means causing to stand. Microbiostatics inhibit the growth of
microbes (c). The suffix -cidal (a) indicates that the agent kills the microorganism (see Degrees of
Microbial Control). The suffix -pathogenic (b) indicates that an agent is a pathogen, not an agent
working against pathogens; and the suffix -biotic means of life.
4. (Answer: a). Whereas damage to DNA would be an example of microbial control, its replication (a) is
not; on the contrary, microbial control seeks to prevent cell replication. Examples of methods of
microbial control include alteration of cell wall permeability (b), oxidation of proteins (c), and
coagulation of proteins (d).
5. (Answer: b). Most methods are more effective as temperature increases (b). Minimum exposure times
(a) set a limit on acceptable times; adhering only to the minimum time does not promote efficacy, and
there are times when the longer end of the suggested time can increase efficacy, depending on the
agent. Some disinfectants are inactivated or affected by organic debris (c) and mixing of disinfectants
can limit their efficacy, make them inactive or cause aversive effects (d).
6. (Answer: d). Addition of 2% calcium carbonate or sodium carbonate (d) will inhibit rust and increase
efficacy. Substituting with sodium chloride, calcium chloride, or sodium bicarbonate (a, b, or c) is not
effective.
7. (Answer: d). Ultraviolet radiation (d) may offer complete sterilization but has no penetrating ability,
whereas dry heat, filtration, and moist heat (a, b, and c) all have penetrating ability.
8. (Answer: c). Cavitation (c) which is the passing of high frequency sound waves passed through a
solution to create cavitation bubbles, cannot provide complete sterilization, unlike incineration (b),
gamma radiation (d), and steam under pressure (a).
9. (Answer: a). High-level products such as aldehydes (a), vapor-phase hydrogen peroxide, chlorine
dioxide, and accelerated hydrogen peroxide are virucidal to both hydrophilic and lipophilic viruses,
as well as being sporicidal. Phenols and alcohols (b and c) are medium-level products, and quats (d)
are low-level products.
10. (Answer: c). Chlorhexidine gluconate or Hibitane is an example of a biguanide (c), usually used as
surgical scrub or a hand wash, providing microbial control through inhibition of cell membranes. It is
not a phenol, quaternary ammonium compound, or halogen (a, b, and d).
11. (Answer: d). Aldehydes, peracetic acid, and ethylene oxide (d) are all considered sterilants. Alcohols
(a), quats (b), and halogens (c) are not classified as sterilants. Peracetic acid and accelerated hydrogen
peroxide are peroxygen compounds (a and c).
12. (Answer: c). Sterilized items must be ventilated in a designated area for 24 to 48 hours to remove
residual ethylene oxide (c). Steam (d), gas plasma (b), and gravity displacement autoclaves (a) do not
require 48 hours of ventilation.
13. (Answer: b). Only gas plasma chambers (b) use the release of free radicals to destroy organisms. Steam
(a) and ethylene oxide (d) do not. With steam (a), heat is the killing agent, and the mode of action with
ethylene oxide (c) is the alkyation of proteins, DNA, and RNA.
14. (Answer: b). Gas plasma is a reliable method to sterilize some plastics and electrical devices (b) that
cannot tolerate high temperatures and humidity. It is not approved for flexible scopes (a) or rigid
scopes (c) with lumens greater than 3 mm in diameter or greater than 400 mm in length. It is not
recommended for strong absorbers (d) and for items that are affected by low pressures.
15. (Answer: b). The minimum effective pressure of the autoclave is (b) 15 lb per square inch (psi), which
provides steam at 250 F (121 C). A minimum of 5 psi (a) would be too low, and 20 (c) exceeds the
minimum. Although many autoclaves attain pressures of 35 psi (d), this is not the minimum.
16. (Answer: d). Complete sterilization of most items is achieved after 9 to 15 minutes of exposure (d) to
250 F (121 C). Anything less than that (a, b, and c) will not achieve complete sterilization.
17. (Answer: b). Dry glass bead (b) does not exist as a quality control method, whereas Bowie-Dick,
serology testing, and chemical indicators (a, c, and d) are all acceptable quality control methods.
18. (Answer: b). Chemical indicators must be placed in the part of the pack that is most inaccessible (b)
before autoclaving. The packs exterior (a) and the center or base of the autoclave (c and d) are all
accessible and thus not good places to put chemical indicator strips.
19. (Answer: b). A thermocouple (b) measures temperatures using sensors that are placed in the part of a
test pack that is most inaccessible to steam penetration. A recording thermometer (a) displays
temperature of autoclave chamber; the operator observes for correct temperature during the cycle,
meaning that it cannot be placed in an inaccessible spot. Chemical indicators (c) are paper strips
impregnated with sensitive chemicals that change color when conditions of sterility are met, and the
Bowie-Dick test (d) evaluates pre-vacuumed autoclaves for complete removal of air and uniform
steam penetration.
20. (Answer: b). The Bowie-Dick test (b) evaluates pre-vacuumed autoclaves for complete removal of air
and uniform steam penetration. Chemical indicators (a) are paper strips impregnated with sensitive
chemicals that change color when conditions of sterility are met. A recording thermometer (c) displays
temperature of the autoclave chamber; the operator observes for correct temperature during the cycle,
meaning that it cannot be placed in an inaccessible spot. A thermocouple (d) measures temperatures
using sensors that are placed in the part of a test pack that is most inaccessible to steam penetration.

CHAPTER 11 COMPANION ANIMAL BEHAVIOR


1. (Answer: d). In extinction (d), a behavior that has a history of being reinforced is no longer reinforced
and goes through the process of extinction. Positive punishment (a) is adding something the animal
finds unpleasant to decrease the likelihood the behavior will happen again. Negative punishment (b)
is withdrawing a pleasant stimulus to decrease the frequency of a behavior. Negative reinforcement
(c) involves strengthening a response or behavior by stopping, removing, or avoiding a negative
outcome or aversive stimulus.
2. (Answer: d). Negative reinforcement involves strengthening a response or behavior by stopping,
removing, or avoiding a negative outcome or aversive stimulus (d). Adding something pleasant (a)
would be positive reinforcement. Adding something unpleasant (b) would be positive punishment.
Subtracting something pleasant (c) would be negative punishment.
3. (Answer: c). At 8.5 weeks (c), canines begin to develop substrate preferences for elimination. Before
that, at 3 to 6 weeks (a and b), they are beginning to concentrate on socialization. At 10 weeks (d) the
developmental focus is on adapting to exploring new environments, as well as an increased capacity
to learn and train. However, its important to be aware that between 7 and 10 weeks, dogs
experience a more fearful period, in which painful or traumatic situations should be avoided.
4. (Answer: c). It is true that the binocular field of view for dogs is poor (c), at only 40 to 60 degrees. On
the other hand, canines have better, not inferior (a) lateral vision than humans, and although their
visual acuity is poor rather than strong (b), they have good motion detection (d).
5. (Answer: d). Aggression is (d) the most common behavioral disorder referred to behaviorists.
Although separation anxiety (a) and fear of loud noises (b) can be problematic, they are not as
common as problems with aggression. House soiling (c) does occur with dogs, but it is the major
behavior problem in cats, not dogs.
6. (Answer: d). Domestic dogs bark for many reasonsincluding all listed in this question (d).
Reasons can include soliciting play or social interaction (a), seeking attention, seeking food,
responding to anxiety (b), or defending territory (c). It may also be a conditioned behavior or result
from cognitive dysfunction.
7. (Answer: b). Whether the cat is marking to signal territory or as an anxiety- or conflict-induced
response, it is best to keep other cats out of the yard (b), because the social system of cats is very
complicated. That means this is also not the time to introduce a new kitten (a). Territorial marking
should never be encouraged, so no, do not let the cat mark the whole house first (c).
8. (Answer: b). Scratching is normal, so the owner should provide scratching posts, which do help
(contrary to choices [c] and [d]) in places convenient for the cat, such as close to where the cat sleeps
(b) and in other prominent areas, as opposed to far away from the animals usual traffic area (a).
This will increase likelihood that the cat will use it.
9. (Answer: d). Skin glands on the head and face are used as scent markers (d); it is thought these
glands are used to mark familiar objects and people within their core territory. Maddening
refers to marking with urine spraying and feces, not scratching (a). It is not true that spreading scent
is always about territory (b); instead, scents are deposited in complex sequences, depending on
message. Finally, it is not true that neutering will put a stop to urine spraying (c). Both intact males
and females, as well as neutered toms will all urine spray.
10. (Answer: a). You will not see a vertical tail (a) in a terrified cat. An upright tail is a sign of a friendly,
relaxed cat. The other choices are all signs of a frightened catpupils dilated (b), ears flattened (c),
and crouched body (d).
11. (Answer: c). Shaping is useful when an animal is performing a behavior in part or performs the
prelude to a behavior, but it has only a small resemblance to the eventual desired behavior, because
steps in the sequence are missing (c). It is not useful in helping an animal who is afraid (a), overeating
(b), or aggressive (d).
12. (Answer: a). Dogs who are conflicted will avert their gaze (a) or look hypervigilantly everywhere. The
other postures are not consistent with anxiety, appeasement, or conflictforward stance (b) (instead,
dogs who are nervous or in conflict will make themselves look smaller or lean away); vertical
retraction of the lips (c) is displayed by confidently aggressive dogs. Finally, ears held erect (d) is a
signal of a confident dog.
13. (Answer: d). Chirring is done by a mother cat calling to her kittens, but it can also be called between
two friendly adult cats (d). Purring expresses contentment, but can occur while a cat is anxious (a).
Meowing can be used to express territory (b), among other things. Finally, a warning of imminent
attack (c) would be expressed as a growl, snarl, or hiss, usually if the animal feels threatened.
14. (Answer: d). All are true (d). Punishment when training any animal has a number of unhelpful side
effects, including (among others) increased aggression or fear, increase in undesirable behaviors, and
inhibition in learning (a, b, and c).
15. (Answer: b). Classical conditioning (b) involves learning an association between two thingsin this
case, carrier = vet visit! In operant conditioning (a), learning would depend on the cats
behavior, not simple association. Counter-conditioning (c) is a method for changing the animals
emotional response. Finally, systematic desensitization is a method for correcting behavior (d).
16. (Answer: c). When putting together a behavioral treatment plan this hierarchy should be followed:
medical, nutritional, and physical needs first (c), followed by management (d), which involves
removing or adding a person or object, based on determination of an antecedent. After that, a history
of the problem and determination of where training lacks (a and b) can form initial steps in creating a
workable planbut only after meeting needs and antecedent management.
17. (Answer: d). The first, and least intrusive, step is to provide plenty of scratching posts and help the cat
avoid areas where previous damage was done (d). After that, reinforcing with treats is fine,
particularly in kittens (b). Punishment should be avoided at all costs, and handling the cat during
scratching will not be comfortable for the cat and may be perceived as punishment (c); spraying the cat
(a) is positive punishment and can have lasting messy side effects.
18. (Answer: b). Setting realistic expectations and avoiding obvious mismatches (b) is the most important
goal of pre-purchase counseling. Unfortunately, it is just not that easy to guide owners into getting the
animal you know would be best (a), tempting though that may be. Most often your clients already
know what they want and will not waver. Adopting a rescue (c) is not for everyone because rescue
dogs can often have some behavior issues already, but rescuing should always be encouraged. Finally,
counseling against adding a second pet to a preexisting pet (d), though it requires some careful
planning, can usually be done with few problems, and can even be beneficial to both animals at times.
19. (Answer: b). A closed-mouth call occurs when a female solicits mating (b). If she were calling to her
kittens (a), she would use a soft chirring sound, and if frustrated (c), she would meow. Finally, a
defensive warning (d) would take the form of a growl, a snarl, or a hiss.
20. (Answer: d). Dogs do not tend to soil out of anger or dominance (d). Instead, it may be due to anxiety,
a medical problem, or lack of training (a, b, and c).

CHAPTER 12 RESTRAINT AND HANDLING


1. (Answer: b). Investigating (b) is the correct answer because cats are curious about their surroundings
and will investigate when introduced to new surroundings. The other behaviors (a, c, and d) are seen
when the cat is upset, scared or sick.
2. (Answer: b). Lowering the ears, hissing, and batting (b) are warning behaviors from an angry cat.
The other distractors are all normal behaviors (a, c, and d) and do not indicate anger.
3. (Answer: a). Wrapping in a towel is an effective way to control an angry cat so that its feet are firmly
enclosed (a). Cats may use the towel for sleeping or hiding (b and c), but these are not effective
restraint techniques, and (d) is not true because a cat can bite and scratch through a towel.
4. (Answer: d). Muzzling a dog with a head injury (d) may obstruct the airway and or make an injury
worse by just applying the muzzle. The other injuries (a, b, and c) will not be compromised by a
muzzle.
5. (Answer: d). Pinning of the ears (d) can indicate an angry or fearful horse. It also is seen when a horse
is concentrating really hard on a task such as reining, jumping, or working cattle. If the horse is alert,
the ears are pricked forward (a); and if the horse is nervous or uncertain, the ears are constantly
moving back and forth (b).
6. (Answer: c). At 6 to 8 feet (c) the power from the kick reaches its full potential, thus causing the most
damage. The distance in (a) is pretty close to the horse, and so the full force of a kick would not be felt.
The distance indicated in (b) is also fairly close, and the power would not be full force. The distance
indicated in (d) is well out of the kicking range.
7. (Answer: a). Twitches are used to distract the horses attention by applying a mild pain to the
upper muzzle (a), which releases endorphins to mask the pain. The other body parts (b, c, and d) are
impossible to put a chain twitch on or will cause extreme damage to the horse.
8. (Answer: d). Cattle have a sensitive nasal septum (d) that can be used to produce a mild pain that
acts as a distraction technique. Pressure can be applied manually; because of the way a nose lead is
designed it cannot be applied to any other parts of the animals face (a, b, and c).
9. (Answer: b). Jacking the tail is done by lifting the tail straight up and forward from the base
(b) so it stands perpendicular to the spinal column. The other distractors (a, c, and d) will damage the
tail or cause the animal to move forward.
10. (Answer: b). Newborn calves are guided in movement by placing one hand under the neck and
placing the other hand around the hindquarters or grasping the tail head (b). The other distractors (a,
c, and d) either will not work, are cruel, or both.
11. (Answer: d). Halter tie (d) is a quick-release knot that is used to tie animals to inanimate objects. The
others (a, b, and c) are not quick release knots. Half hitch (a) needs a series to make it secure; tomfool
knot (b) is a version of the square knot, and the bowline (c) will make a nonslipping noose.
12. (Answer: b). Sheep (b) have strong flocking instincts and move as one as a protection mechanism.
Horses, goats, and pigs (a, c, and d) will scatter when chased.
13. (Answer: a). Mare and foal (a) is the correct answer. A mare will fret and fight to get back to her baby,
and so the foal needs to be worked on in line of site. Piglets need to be taken out of sows hearing
(b) before being worked on. Lambs and kids (c and d) can be either taken away or left; the mothers
will vocalize but not break down fences to get to their young.
14. (Answer: d). Sheep (d) wool is very thermodynamic, and they overheat even in mild temperatures.
The other animals (a, b, and c) will also overheat in extremely not weather but not just by walking or
minimal activity.
15. (Answer: c). The tendency to pull back in the opposite direction (c) is the correct answer. Because pigs
often will do the opposite of what you want, we use that to our advantage by applying the hog snare
over the snout and leaning or pulling back. They pull in the opposite direction. It has nothing to do
with the snouts strength or sensitivity (a and d), and though they are curious they are not always
easy to catch (b).
16. (Answer: c). In the sternal position (c) the restrainer can push the leg forward, control the head, and
keep the animal from backing upall with just two hands. Dorsal recumbency (a) with the animal on
its back and lateral recumbency (d) on its side would make it very difficult to reach the front leg.
Standing (b) really does not allow for much control. You can begin with the dog in standing restraint,
but then immediately place in sternal position.
17. (Answer: a). A muzzle that completely closes the mouth should not be left on longer than 20 minutes
(a) without a break. Anything longer than that (b, c, or d) is uncomfortable and can possibly cause
injury. If using a cage muzzle that allows panting, that time expands to many hours.
18. (Answer: c). Beef cattle (c) in particular are run through a chute for these and other procedures. Sheep,
pigs and horses (a, b, and d) are either moved into a pen (a), captured with a hog snare (b), or haltered
and secured (d).
19. (Answer: d). As with dogs, cephalic venipuncture requires the cat to be in sternal recumbency (d).
Lateral recumbency (a) has the cat lying on its side, there is very limited control with standing (c), and
dorsal recumbency (b) is not commonly used.
20. (Answer: a). Slow, deliberate movements (a) are preferred to prevent startling the animals. The other
choices are false because (b) a low command to move is much preferred over sharp yelling; (c) do not
approach from the front because it is a natural instinct for cattle to charge. Contrary to choice (d), cattle
cannot charge if they are in a squeeze chute and head gate, or tied in a stanchion.

CHAPTER 13 SMALL ANIMAL NUTRITION


1. (Answer: b). Taurine (b), along with arginine, is a requirement in feline diets to avoid deficiencies.
Also, taurine is the only amino acid listed in this group; carnitine (a) is not an amino acid but a
compound biosynthesized from the amino acids lysine and methionine, and both niacin and thiamin
(c and d) are B vitamins.
2. (Answer: d). Based on the new AAHA and WSAVA nutritional guidelines, a nutritional assessment
should be done at every visit of the pet (d), not just on sick or overweight cats (b and c). Annually (a)
is not often enough if there are health or other issues between the annual examination.
3. (Answer: c). At week 5, energy needs increase markedly. The period to begin transition to a high-
quality, highly digestible growth diet should be during the last 3 to 4 weeks of gestation for a dog and
from the second week of gestation (b) in the queen. During week 1 (a), fetal tissue development is
minimal until about day 40, so transition is not yet recommended. Week 9 or the last week of
gestation is too late (d).
4. (Answer: b). Choice (b) 550 kcal/day is correct. For an adult dog needing weight loss, MER is 1.0
RER. RER = 30 (Weight in kg) + 70; then MER = RER 1.0 (to achieve healthy weight
loss). Final calculation is: RER = (30 16 kg) + 70 = 550, MER 550 1.0 = 550
kcal/day (correct). Choice (a) uses the 0.8 MER factor, which is recommended for overweight cats, not
dogs; the factor should be 1.0 instead. Choice (c) 1792 kcal uses the pounds instead of the kilograms in
the formula. Choice (d), 461 kcal is incorrect.
5. (Answer: d). All of these are true (d). Fiber is beneficial because of its ability to absorb water (a),
helping prevent water loss and diarrhea; its ability to stimulate intestinal contractions (b); and to
normalize intestinal transit time (c).
6. (Answer: d). Hematuria (d) is common because of inflammation of the bladder and crystal irritation.
Cats with FLUTD may appear to be at the litter box defecating, but it is generally frequent urination,
not defecation (a). Increased hunger (b) is not a common clinical sign, and although weight gain (c) is
a risk factor for FLUTD, it is not a clinical sign.
7. (Answer: d). An obese animal is one that weighs more than 20% (d) of its ideal body weight. Animals
that are 5% to 15% overweight do weigh more than the ideal but are not considered obese (a, b, and
c).
8. (Answer: b). A rate of 0.5% to 1% of the obese body weight per week (b) is the safest rate of weight
loss in cats. A rate of 0.5% to 1% of the ideal body weight per week (a) would result in too rapid of a
weight loss. The rate of 1% to 2% of the obese body weight per week (c) is the safest rate of weight
loss for dogs, not cats. The calculations are not based on a rate of 1% to 2% of the ideal body weight
per week (d).
9. (Answer: b). There is no nutritional adequacy statement by AAFCO for senior and mature life stage
(b) at this time. The four life stages that are recognized are all life stages (a) (puppy/kitten), gestation
and lactation (b), adult maintenance (d) and growth.
10. (Answer: d). In dry weight analysis (d), converting nutrients to dry matter allows for a more accurate
comparison of products with different moisture levels. A guaranteed analysis (a) represents only
minimum or maximum and includes moisture. In the case of ingredient panels (b) there is no way to
evaluate the nutritional quality of ingredients listed. Finally, the nutritional adequacy statement (c)
only identifies whether the product meets nutrient profiles for cats and dogs based on feeding trial or
calculated method.
11. (Answer: a). Excess protein will be burned for energy and can provide 4 kcal/g (a) if energy is not
obtained from carbohydrates or fat. The values 6 and 7 (b and c) are fabricated. Fat provides the
most concentrated source of energy at 9 kcal/g (d).
12. (Answer: a). Guaranteed analysis (GA) on a label indicates that the manufacturer claims the product
meets either minimum or maximum percentages of certain nutrients (a). Federal law requires that the
ingredients be part of the label (b) but they are not part of the GA. Exact percentages or ratios of
certain nutrients (c and d) are not required.
13. (Answer: c). The range of water in dry kibble is small10% to 12% (c). Less than that makes bonding
impossible (a and b), and with 20% moisture (d) it would no longer be dry kibble.
14. (Answer: a). Lactose content in cows milk is greater than that of a bitch (a); this may cause
diarrhea and dehydration. Although it is possible that there may be less calcium than needed for
puppy growth (b) and too much fat (d), neither of these is nearly as harmful as the level of lactose.
Although puppies may lose weight by refusing to drink cows milk (c), this too is not nearly as
harmful as the higher levels of lactose.
15. (Answer: b). In feeding large-breed puppies, the main goal is to (b) decrease the growth rate but still
reach the dogs genetic potential at maturity. This will ensure proper skeletal development. By
contrast, increasing the growth rate (a) can negatively affect bone volume, bone modeling, and
cartilage maturation. Free choice feeding (c) will increase the risk for obesity. Finally, feeding them
adult dog food (d) would cause puppies to miss the higher nutrient requirements they need for
growth.
16. (Answer: d). The main disadvantage of free choice feeding is that it is difficult to monitor the pets
consumption (d) and thus may lead to obesity and nutritional excesses. The other three choices (a, b,
and c) are not disadvantages, but advantages.
17. (Answer: b). Growth diet can be introduced at 3 weeks of age (b) as a slurry or in canned or dry form
depending on what they will accept. Two weeks (a) is a bit young to offer the food; and 5 or 7 weeks (c
or d) is a bit late to start introducing real food since they should be already weaned at 7 weeks.
18. (Answer: d). All of these factors (d) should be applied in elimination diet trials. They are used to
identify allergens (c); they should be used for approximately 4 to 10 weeks (a), which allows for the
immune system to adjust to the change in ingredients. These diets contain a limited number of novel
protein sources or a protein hydrolysate (b), which helps identify or eliminate the allergen.
19. (Answer: a). The dry matter conversion allows for a more accurate comparison of fat, protein, or
carbohydrate levels (b, c, and d) that is not consistent when displayed at different moisture levels (a).
Manufacturers should provide nutrients listed on a DMB for comparison and accurate values of the
fat, protein and carbohydrate levels (b, c, and d).
20. (Answer: a). It is true that cats use a certain amount of protein for energy (a) because cats are
carnivores, not omnivores (b). Although cats do tend to live longer than dogs (c), this is not the reason
for requiring more protein. Whether they eat more or fewer small rodents and birds than dogs (d) has
no bearing on their inborn protein requirements and is more a reflection of life circumstance and
opportunity.
CHAPTER 14 LARGE ANIMAL NUTRITION AND FEEDING
1. (Answer: a). Soybean meal, linseed meal, and cottonseed are commonly fed as a protein supplement.
Common grains such as oats and barley (b and c), legumes, and grass hay (d) are the base constituents
to a feed ration and may not provide adequate protein levels for growth, reproduction, and lactation.
2. (Answer: d). Horses and ponies genetically predisposed to obesity are at risk for equine metabolic
syndrome. Equine metabolic syndrome is not related to reproductive, nutritional, or muscle
inflammatory disorders (a, b, and c).
3. (Answer: c). Urea (c) is a nonprotein nitrogen that can be used by ruminants as an economical
replacement for some protein in a feed ration. Neither starches, sugars, nor fiber (a, b, and d) can
serve as replacements for protein.
4. (Answer: b). Ruminants have an increased risk for acquiring the protein that causes spongiform
encephalopathy. Certain mammals (b) are the source for this protein. It is not a risk in birds, reptiles,
or aquatic animals (a, c, and d).
5. (Answer: d). Calves require at least 10% of their bodyweight (d) of high-quality colostrum to
maximize immunoglobulin absorption. Any amount less than that (a, b, or c) is insufficient to
maximize absorption of immunoglobulins.
6. (Answer: b). Lambs and kids benefit from creep feeds (starting at 2 weeks (b) to ensure maximum
growth rates. Feeding them before that time (a) is ill-advised, and waiting past that (c and d) does not
provide maximum growth opportunity. Creep feeds are ground coarse or rolled grain, and hay that
their dams do not have access to.
7. (Answer: d). Camelids grow a thick fiber coat (d) that tends to obscure the areas that would be
assessed to determine body condition. Spitting does not make them impossible to approach (a), nor
does kicking (b). The amount of bulk along the backbone does help provide a clear objective means of
evaluating body condition, making choice (c) false.
8. (Answer: c). Although swine are more commonly fed concentrates for production, they can
accommodate some dietary fiber, which makes swine omnivores (c), meaning that they can eat both
meats (a) and vegetable matter (b); eating insects (d) is of no particular value to swine.
9. (Answer: a). Fats provide dietary energy, a source of heat, insulation, and body protection (a) and
serve as a carrier for absorption of fat-soluble vitamins. Although they provide some dietary energy,
they do not contribute to growth and reproduction, tissue repair, or waste removal (b, c, and d).
10. (Answer: c). The omasum (c) is filled with muscular laminae or leaves that squeeze fluid from
the ingesta as it passes to the abomasum (d) or true stomach. The reticulum and rumen ([a] and [b])
are forestomachs as well, but are cranial to the omasum and abomasum; thus digestive processes take
place in these before they are passed into the omasum: only the omasum connects into the abomasum.
11. (Answer: b). Cattle that consume pastures that are deficient in magnesium (b) are prone to grass
tetany (hypomagnesemic tetany). Lack of selenium, copper, and fluoride (a, c, and d) do not cause
grass tetany.
12. (Answer: a). Imbalanced diets and evidence of mycotoxins or poisonous plant poisoning can lead to
thiamine deficiency (a) polio (polioencephalomalacia) characterized by cerebrocortical necrosis.
Polioencephalomalacia is not caused by a deficiency of riboflavin, folic acid, or niacin (b, c, or d).
13. (Answer: d). Milk normally bypasses the rumen after closure of the reticular groove (d), allowing milk
to travel directly to the abomasum; this direct connection is not made through the pharynx,
esophagus, or pylorus (a, b, or c). Milk that remains in the undeveloped rumen tends to curdle,
leading to indigestion. Allowing the calf to suckle before or during feeding allows adequate closure of
the reticular groove.
14. (Answer: b). Red clover, birdsfoot trefoil, and alfalfa (a, c, and d) are classified as legumes, not grasses,
like timothy (b).
15. (Answer: c). A sows teat line can easily be contaminated with fecal material, which contains
microbial pathogens that can cause scours (c) in piglets, but not anemia, parakeratosis, or water belly
(a, b, or d).
16. (Answer: d). Enzymatic digestion occurs in the small intestine (duodenum, jejunum, and ileumthat
is, a, b, and c); fermentation takes place in the cecum (d) and large intestine.
17. (Answer: a). Rice, wheat bran (a), and beet pulp are fermentable fiber by-products fed to horses.
Wheat bran has been used to increase fiber bulk. Oats, corn, and mineral oil (b, c, and d) do not
contribute to preventing or treating sand colic.
18. (Answer: a). Proteins (a) are essential for the growth and development of young horses.
Carbohydrates and fats (b and c) provide energy, and electrolytes (d) aid the bodys metabolic
processes.
19. (Answer: c). Dusty feeds can lead to respiratory disease in animals. Adding moisture (e.g., molasses,
water) (c) to the dusty feed can improve palatability and reduce or eliminate dust-related respiratory
signs. Neither salt, fiber, nor sodium bicarbonate (a, b, or d) will help reduce the risk for respiratory
effects caused from dusty feeds.
20. (Answer: d). Salt (d) can be fed free choice as long as water is readily available. Calcium, phosphorus,
and selenium (a, b, and c) must be carefully balanced in a ration.

CHAPTER 15 GENETICS, THERIOGENOLOGY AND NEONATAL


CARE
1. (Answer: b). Outbreeding (b) maintains the largest gene pool by extending relations farthest beyond
related blood lines. Both inbreeding and line breeding (a and d) involve closely related individuals,
limiting the size of the gene pool. Harem breeding (c), while involving several females, involves only
one male, again limiting the genetic diversity.
2. (Answer: a). Epistatic genes (a) are those that modify or prevent the expression of other genes.
Polygenic (b) describes traits that are affected by multiple genes. Dominant genes (c) are those that are
always expressed when present. Lethal genes (d) are those that will cause the death of the embryo or
else cause serious impairment or death sometime after birth (when it is sometimes called a delayed
lethal gene).
3. (Answer: b). Phenotype (b) describes the manifestation of the genotype, (e.g., outward appearance,
behavioral characteristics, metabolism). Alleles (c) are the forms of a gene that may be present. A
genotype (a) describes the alleles an organism possesses. Finally, a chromosome (d) is a component of
an individuals DNA; genes are segments of that single DNA chromosome that provide the code
for specific proteins or regulate the expression of other genes.
4. (Answer: d). Phenotypes of offspring produced are in the ratio of 9:3:3:1 (d). The other ratios (a, b,
and c) are in the wrong order.
5. (Answer: d). In cats (d), ovulation is not spontaneous; instead, ovulation occurs only when the animal
is mated or the cervix is stimulated by an instrument, such as a glass rod, cotton swab, or
thermometer. It can also be induced by treatment with hormones. The other species have naturally
occurring estrus and ovulation cycles. Horses (a) are seasonally polyestrous, with ovulation occurring
1 to 2 days before the end of estrus. In swine (b) ovulation occurs on the second day of estrus, and in
dogs (c) it occurs 2 to 4 days after onset of estrus.
6. (Answer: a). The prostate gland (a) is the only accessory gland in the dog. The canine preputial gland
(c) is not discussed in this chapter. Finally, the other two structuresthe seminal vesicles and the
Cowpers gland (b and d) are accessory glands in the stallion.
7. (Answer: c). Three speciesbovine (c), ovine, and caprine (not listed)have cotyledonary
placentation, whereas the equine species (b) have both diffuse and microcotyledonary placentation
(see Table 15-1). The feline species (a) have zonary placentation, and porcine species (d) have diffuse
placentation.
8. (Answer: b). Dogs (b) have no true estrous cycle, but experience estrus twice yearly. Cats (a) are
seasonally polyestrous; cows (c) are nonseasonally polyestrous (7 to 18 months); and, finally, the
equine (d) estrous cycle is usually 20 to 33 days, with estrus being 5 to 6 days, and is divided into
follicular and luteal phases.
9. (Answer: d). Cows (d) are nonseasonally polyestrous (7 to 18 months). The feline and caprine species
(a and b) are both seasonally polyestrous. Finally, the equine (c) estrous cycle is usually 20 to 33 days,
with estrus being 5 to 6 days, and is divided into follicular and luteal phases.
10. (Answer: c). In cows, ovulation occurs 12 to 18 hours after end of estrus (c). Ovulation occurs 1 to 2
days before end of estrus (d) in horses. In swine, ovulation occurs on the second day of estrus (a), and
it occurs 24 to 30 hours after onset of estrus (b) in both caprine and ovine females.
11. (Answer: c). In canines, the diestrus (c) cycle is characterized by an abrupt decrease in the number of
cornified epithelial cells, along with whelping from day 1 of diestrus. In proestrus (a) noncornified
squamous epithelial cells predominate in vaginal mucosa; followed by cornified squamous epithelial
cells in vaginal mucosa during estrus (b). Finally, by approximately the tenth day after estrus, all
epithelial cells in the vaginal mucosa are noncornified (it is difficult to differentiate between anestrus
and metestrus) (d).
12. (Answer: a). In canine proestrus (a), noncornified squamous epithelial cells predominate in vaginal
mucosa; followed by cornified squamous epithelial cells in vaginal mucosa during estrus (b). In
canines the diestrus (c) cycle is characterized by an abrupt decrease in the number of cornified
epithelial cells, along with whelping from day 1 of diestrus. Finally, by approximately the tenth day
after estrus, all epithelial cells in the vaginal mucosa are noncornified (and it is difficult to differentiate
between anestrus and metestrus) (d).
13. (Answer: a). The use of promazine tranquilizer can cause penile paralysis in stallions (a). No mention
is made in the text of this drugs effects in the bovine, ovine, or porcine species (b, c, or d).
14. (Answer: c). Caprine gestation lasts an average of 149 days (c). Bovine gestation is approximately 278
(a) days and equine 336 days (b). Porcine gestation lasts approximately 114 days (d).
15. (Answer: c). Flushing occurs in ewes (c), not in the other species listed here (a, b, or d). Increasing the
dietary intake before ewes are bred can increase the number of follicles that mature and rupture.
16. (Answer: a). Q-fever (a) is caused by a rickettsial organism, Coxiella burnetii. It has zoonotic potential
and causes abortion, particularly in ruminants. Pasteurella haemolytica pneumonia and coccidiosis (b
and d) are more commonly diseases in young kids, and toxoplasmosis (c) is not discussed in this
chapter.
17. (Answer: a). Vocalization (a) is not a sign of estrus in horses. Instead, signs of equine estrus include
squatting, vulvar winking (b), tail raising (c), and urination (d).
18. (Answer: c). The ovine species (c) has an estrus cycle of approximately 17 days; the bovine, caprine,
and porcine cycles (a, b, and d) all last approximately 21 days.
19. (Answer: c). The soonest radiographs can determine pregnancy is 43 days (c), long before (d) 62 days.
Ultrasound, not radiography, can determine pregnancy at 24 days (a) or even sooner; and at 28 to 30
days (b) hormone assays can determine pregnancy.
20. (Answer: d). Among horses (d), the average age of puberty is 12 months12 months in females and
12 to 18 months in males. In cats (a) the average is 9 months; in dogs (b), the average is 8 months.
Swine (c) reach puberty between 4.5 and 6 months in females and 5 to 7 months in males.

CHAPTER 16 LABORATORY ANIMAL MEDICINE


1. (Answer: a). The rabbit (a) has two pairs of upper incisors with a smaller pair behind the larger pair.
The other species, guinea pig, gerbil, and hamster (b, c, and d), have only a single pair of incisors.
2. (Answer: c) The rabbit (c) practices coprophagy out of nutritional necessity, while in other species (a,
b, or d), coprophagy might occur but would more likely be induced by boredom or stress-related
factors.
3. (Answer: c). The guinea pigs and the nonhuman primates (c) both require supplemental vitamin C,
because both lack the enzyme responsible for synthesis. The others listed hererabbit, degu, and
hamster (a, b, d) do not suffer vitamin C deficiencies and can synthesize needed nutrients.
4. (Answer: b). Good hygiene for all animals of suspect health status (b) is the best of the strategies
listed here. There are no known vaccinations to prevent this viral infection (c), and antibiotic therapy
(a) is not effective against a viral infection. The method of burning out a viral infection by
ceasing to breed so that the infection runs its course (d) or keeping animals under microisolation is
known to be effective only with MHV infections, and has not been proved with LCMV.
5. (Answer: c). The female will have a postpartum estrous cycle within 24 hours after parturition (c).
This means that when the current litter of pups is just 19 to 21 days of age, she will have a newborn
litter. This must be considered, because many strains of mice benefit from a 28-day weaning. Males
are not known to cannibalize the litters (a). The female will not be pseudopregnant (b); she will be
actually pregnant in the constant presence of a male. Males will not disturb the nest (d), and in fact
will take turns keeping the pups warm.
6. (Answer: b). It is documented that 20 (b) consecutive brother and sister or father and daughter
matings in mice will result in an inbred line. Ten (a) may not produce an effective inbred line,
whereas more than 20 (c and d) only compounds risks.
7. (Answer: d).Clostridium piliformis (d) is the only known bacteria to cause Tyzzers disease. None
of the others (a, b, or c) have been associated with the disease. Sialodacryoadenitis virus (a) is a
coronavirus, not a bacterium. Lawsonia intracellularis (b) causes proliferative ileitis or transmissible
ileal hyperplasia. Mycoplasma pulmonis (c) is thought to contribute to other disorders such as
respiratory infections, but not Tyzzers disease.
8. (Answer: a). Rats and gerbils (a) are the only pair of animals listed that have a harderian gland
behind their eyes. This gland secretes a red porphryin secretion that is rich in lipids and proteins and
acts as a lubricant for the eye. Guinea pigs, chinchillas, hamsters, and mice (b, c, and d) do not have
this gland.
9. (Answer: c). Guinea pigs should be bred before the age of 6 months (c), or there is an increased risk
for pubic symphysis, which can inhibit passing of the fetus through the birth canal and result in
dystocia. Guinea pigs are too young to breed at 2 months (a) but can be bred at 3 months as there is no
concern for pubic symyphysis (b), whereas not allowing breeding until 9 months (d) is too late.
10. (Answer: a). Guinea pigs and chinchillas (a) are born precocious with open eyes and full hair coat. All
the other species listed (b, c, and d) are born hairless and quite reliant on parental support.
11. (Answer: b). The barber (b) will have all its whiskers, whereas the others in the cage will have
had their whiskers trimmed off. If this was due to an infection of any kind (a, c, or d), most likely all
animals would be affected and be showing some symptoms.
12. (Answer: b). Sentinel programs (b) are designed to check for all pathogens of concern in a rodent
population. The sentinel is directly exposed to other resident animals or to some of the soiled bedding
collected during cage change and will mount an immune response or titer or will have become
infected and be harboring the organism in body tissues. Personal protective equipment (d) may assist
in the management of the outbreak, as would microisolator caging (a), but are not part of the screening
process. Most of the mouse pathogens are latent infections and do not overtly cause disease, therefore
full physical examination and radiographs (c) are not that useful for health screening
13. (Answer: a). Constantly erupting (a), or hypsodontic, teeth are common in laboratory animals and
require supervision to ensure that they do not overgrow. The gland behind the eye in the gerbil and
rat (b) is the Harderian gland. Hypsodontic does not refer to immobilization technique, or bone
marrow type (c, or d).
14. (Answer: b). The gestation period of a mouse is 19 to 21 days (b). All other ranges (a, c, and d) may
seem close but are not accurate.
15. (Answer: b).Herpesvirus simiae (b) is a B virus that is highly pathogenic to humans and results in
encephalitis with high morbidity and mortality. The other bacteria and viruses listed (a, c, and d) are
not known to infect nonhuman primates.
16. (Answer: c). Only chinchillas (c) require a frequent dust bath, at least twice per week. They will dust
bathe up to 1 hour. The other species (a, b, and d) do not have this requirement.
17. (Answer: a).Salmonella, ringworm, and lymphocytic choriomeningitis (a) are all zoonotic.
Chromodacryorrhea is not; instead, this disease is seen as a by-product of stress and anxiety. Thus b, c,
and d are incorrect. Scurvy is not either (d).
18. (Answer: c). Hamsters (c) are used in hypothermia studies because they are able to go into short
periods of pseudohibernation when temperatures are under 48 F (10 C) and daylight hours
shorten. The other species, gerbils, guinea pigs, and mice (a, b, and d) do not share this ability.
19. (Answer: d). Pasteurellosis caused by the bacteria Pasteurella multocida bacteria is treated with
antibiotics, so to deny this (d) is false. The other answers are all true. Pasteurellosis can be passed
either directly or through fomites (a); its manifestations include rhinitis, torticollis, abscesses, and
vaginal discharge (b); and its signs can be suppressed without the organism itself being eliminated (c).
20. (Answer: b). Chromodacryorrhea is a bacterial condition (b) often caused by Staphylococcus spp. It is
not a viral condition (a), caused by porphyrin secretions from the harderian gland (c) or caused by a
secretion from the hibernating gland (d).

CHAPTER 17 EXOTIC ANIMAL MEDICINE


1. (Answer: c). Vaccinations in ferrets are primarily against rabies and canine distemper (c). Ferrets
should be vaccinated yearly, ruling out (d). Nothing is said about vaccinating ferrets against either
feline distemper or mink parvovirus (a and b).
2. (Answer: b). Due to increased insulin from the pancreas, it is the most common neoplasia (b);
antibiotics or pain relief are not relevant to insulinoma (c); decreased insulin will not cause
insulinoma(a); it is common in ferrets so this is not correct (d).
3. (Answer: c). Hyperadrenocorticism in ferrets is due to sex hormones (c) and not to cortisol, as seen in
Cushings disease in dogs (a and b); contrary to choice (d), it is seen in older neutered ferrets.
4. (Answer: b). Degus can develop spontaneous diabetes mellitus (b). Ferrets get hyperadrenocorticism
and lymphoma (a and d); Squamous cell carcinoma (c) is commonly found in hedgehogs.
5. (Answer: d). Neither coulter counter machines nor EDTA should be used (d). Coulter counter
machines will incorrectly count the nucleated RBCs as white blood cells (a). The blood should be in
heparin tubes (b) because EDTA (c) can lyze these cells.
6. (Answer: c). Anatomy of the GI tract is such that the crop is before the proventriculus, ventriculus,
and cloaca (c). The ileum is distal to proventriculus (ruling out a); ventriculus is part of the
stomach in birds (ruling out [a], [b], and [d]); the esophagus is before crop (ruling out [a] and
[d]) and duodenum after ventriculus (ruling out [d]).
7. (Answer: b). High-risk birds for anesthesia include parrots with a crop distended with fluid (b).
Crops filled with fluid can be cause of regurgitation leading to aspiration. Beak size (a), design or
abnormalities (d) are not mentioned as concerns for anesthesia. Anesthesia would be required to
repair a fractured leg (c).
8. (Answer: a). The air sacs are along the back of birds, so to improve respiratory efficiency under
anesthesia, birds should be kept in ventral recumbency; therefore if lying on their backs (dorsal)(b), all
internal organs would compress and not allow the bellows system of the sternum to work properly,
making respiration more difficult. So ideally they need to lie in sternal recumbency or on their side as
the abdominal organs do not affect the air sacs. The sternum (c) does not affect ventilation and the air
sacs are best inflated in ventral so (d) is incorrect.
9. (Answer c). Birds can have eight or nine air sacs; the humerus and femur are listed as proximal,
pneumatic bones where air sacs connect (c); The radius, ulna, and tibia are too far, or distal, from the
main part of body to allow connection of air sacs (a, b, and d).
10. (Answer: d). Nucleated RBCs are found in fish and in all birds and reptiles (d), not just in parrots and
tortoises (b and c); they are not present in mammals (a).
11. (Answer: d). Uric acid excretion is mentioned for both birds and terrestrial reptiles only (d)but not
all reptiles (a). Aquatic species of reptiles produce some urine too, but this answer excludes birds (b).
Obviously, then, birds are not the only animals to excrete uric acid (c).
12. (Answer: c). Reptiles do require the UV light source for conversion (c). Reptiles do benefit from a
basking or heat lamp to provide thermal gradient, but not through a heated substrate on the cage floor
(a), and neither heat nor a basking rock are required for vitamin D synthesis and calcium absorption (a
and d). Humidity and temperature control are important, but not for vitamin D synthesis and calcium
absorption.
13. (Answer: c). Birds and reptiles (c) both have a renal portal system. This system is not present in
mammals (b, and d). Turtles (a) are not the only reptiles that have a renal portal system.
14. (Answer: c). To restrain a lizard, grasp behind the head at the mandible and hold the pelvis and tail
base with the other hand (c). The tail can break off if held in some species (a), and the carapace is only
present in turtles (b).
15. (Answer: b). The terms ecdysis and dysecdysis refer to shedding of skin (b); dys- is a prefix meaning
difficult. This is a condition relating to skin and not thermoregulation, parasitic conditions, or
defecating (a, c, or d).
16. (Answer: b). Lizards and chelonians possess a tympanic membrane, whereas snakes do not (b); snakes
do not possess a middle ear cavity (c), so sound is not a major receptor for them (d). Instead, some
have infrared receptors that are extremely sensitive to changes in heat; the receptors also help
determine distance and direction in finding prey; all reptiles do not lack a tympanic membrane (a).
17. (Answer: c). Lizards and chelonians have 2 saccular lung(s) and snakes have one lung on the right side
as the left side is reduced or absent. The other answers (a, b and d) are not correct.
18. (Answer: b). Metabolic disease occurs as a result of low calcium and UVB lighting (b), not high
calcium or UVA lighting or genetics (c and d). It can occur in young or old reptiles (a). It is treated
with calcium therapy, broad-spectrum ultraviolet light supplementation, and diet correction.
19. (Answer: c). Two methods used to anesthetize fish are MS-222 and metomidate (c). The drug is not
referred to as MS-22 (a) or MDS-222 (d). Medetomidine (b) is available for use in small animals but is
not used in fish.
20. (Answer: d). To obtain a blood sample from a fish, you would use the caudal vein (d). The other veins
are not accessible or present (a, b, and c)

CHAPTER 18 ZOONOSES
1. (Answer: a). Cat scratch fever is transmitted by domestic cats in normal oral flora and transmitted to
claws during grooming (a). It is therefore zoonotic, contrary to choice (d). It is bacterium and not a
rhabdovirus (b). It is not serious (c) and is usually self-resolving.
2. (Answer: b). Ancylostomiasis, or hookworm, can be acquired if larvae work their way into the skin of
a person or animal (b). For this reason, it is not a good idea to keep childrens sandboxes
uncovered (d); uncovered sandboxes invite cats and wildlife to use them as a litterbox and increases
likelihood of hookworm infection. Hookworm is not detected using interdermal tuberculin testing,
radiography, or blood culture and isolation (c), nor is it very difficult to detect (a). Instead, it is easily
diagnosed by sighting ova on fecal flotation, as well as through clinical presentation and appropriate
history.
3. (Answer: d).Leptospira is primarily transmitted by contact with infected urine, water, soil, or animals
(d) and not by fecal-oral contamination, bites from infected birds, or during molting (a, b, or c).
4. (Answer: b). Mycobacterial infection is detected using interdermal tuberculin testing, radiography,
and culture and isolation (b). It is not true that it is contracted by only a few species (d) or that it can
be controlled through childhood vaccination programs (c). Finally, it is not known to cause hepatic
dysfunction but is associated with the respiratory system, primarily causing tuberculosis (a).
5. (Answer: d). All of the statements made here (d) about the pathogen Toxocara spp. are true. It can
cause visceral larval migrans (LVM) and ocular larval migrans (OLM) in humans (a), as well as
intestinal obstruction in the host (c). The pathogen can infect most species of animals, as well as
humans (b).
6. (Answer: a). In humans, MRSA often initially manifests as a small pimple or boil (a). It is not true that
exposure always results in a pathological condition (c); instead, some individuals do not become
infected but become colonized by the bacteria. It is also not true, then, that it always results in
amputation of the infected limb (b); by contrast, the truth is that many infections can be successfully
treated with antibiotic therapy. Finally, it is not true that MRSA is difficult to transmit (d);
unfortunately, transmission can occur very easily by direct contact or environmental contamination.
7. (Answer: a). Tularemia (c) is the only disease listed here that is not transmitted by rickettsial
organisms. The othersRocky Mountain spotted fever, Lyme borreliosis, and Ehrlichia (b, c, and d)
are.
8. (Answer: c). Tapeworm infections may be transmitted by eating an infected flea (c). It is not true that
they are transmitted when the tapeworm segments break off and grow into a new, fully formed adult
or that they are always highly pathogenic (a or b). Finally, they cannot be treated with antibiotics (d);
instead they must be treated with cestode-specific anthelmintic medication such as praziquantel.
9. (Answer: a). To reduce the exposure to, and transmission of, avian Influenza type A viruses (i.e.,
subtypes H5N1, H5N2, H7N3, H7N7, H7N9), use appropriate personal protective equipment
(a)masks and hand hygiene are the best prevention. The use of cold water over heat or chemical
compounds (c) is false; in fact, the opposite is true: disinfection, heat sterilization, alcohol, 5% bleach
solution, formalin, and iodine compounds are effective disinfection methods. Long-term storage of
dead, possibly infected birds on poultry farms for future study and testing purposes (d) is not a good
idea. Instead, immediate removal and disposal of dead birds is required. Finally, of course attending
markets and farms where live poultry are housed or sold (b) is false.
10. (Answer: c). The avian species (c) are the primary reservoir for Chlamydia psittaci, not mammals or
reptiles (a, b, or d).
11. (Answer: c). Dermatophytosis (ringworm) infections are primarily acquired by contact with
contaminated skin, hair, equipment, bedding, or contaminated soil (c), not via the fecal-oral route (d).
They are not parasitic (a), but mycotic (fungal); the lesions are not black, flat, and linear (b), but red,
raised, and circular.
12. (Answer: c). Incredible as this may sound, the rhabdovirus that causes hydrophobia has an incubation
period of 9 days to 2 years or longer (c). That is because the virus migrates through the body very
slowly. If the bite is on a distal part of the body, the incubation period may be very prolonged; as long
as 8 to 9 years has been reported. It is not true that there are multiple successful treatments for this
disease (a); there is no effective, reliable treatment for rabies. Behavioral changes can begin during
stage 1, not stage 2 (b), which is characterized by progressive paralysis and death. Finally, it is not true
that there is a simple laboratory test to identify this infection (d); instead, the only tests that are
available involve tests that are run on brain tissue (hippocampus, brain stem, cerebellum).
13. (Answer: d). All of these statements are true (d) about salmonellosis transmission. It is true that
washing carriers such as reptiles and their cage items in the kitchen sink (a) increases the chance of
oral ingestion of the bacteria and subsequent infection, as does using dog chew treats made from
swine ears or snouts (b); and feeding a BARF diet to pets (c); raw meat diets have a marked increase of
salmonella ssp. contamination.
14. (Answer: d). The primary vector for the West Nile Virus is mosquitoes, primarily the Culex ssp. (d),
not the fruit bat, crocodile, or wild avian species (a, b, or c), the latter of which serve as reservoirs, but
not vectors.
15. (Answer: c). In scabies, microscopic examination of skin scraping will show mites and ova from
infected lesions (c), even though recovery of these may be difficult because of the thickened lesions.
These lesions not only thicken but also are alopecic and pruritic in both humans and animals, contrary
to (a). It is not true that animals are not pruritic, as asserted in choice (a); instead, patients can receive
prophylactic ivermectin. Finally, the mites that cause scabies are not transmitted by air (d) but by
direct contact.
16. (Answer: d). Alveolar hydatid disease manifests as jaundice, epigastric pain, and hepatomegaly (d). It
may also include malaise and hepatic cysts. It is not true that their definitive hosts are prey species (b);
instead, the prey species are intermediate hosts; predators are the definitive hosts. The proglottids are
not large (a) but rather very small. Finally, treatment in humans requires excision of the cysts, not
anthelmintic therapy (c), which is used to treat animals only.
17. (Answer: a). Swine are, it is true, susceptible to both human and avian influenza viral infections (a).
However, it is rarely fatal as in (b), and in fact many infected swine never show signs of illness.
Transmission is not primarily fecal-oral (c), but is primarily through droplet inhalation and direct
contact. Rapid flu tests (d) often show a false-negative result.
18. (Answer: b). Toxoplasmosis is caused by a single-celled microorganism (b). It is not caused by
nematodes, mites, or fungal spores (a, c, or d).
19. (Answer: a). The Yersinia spp. that causes the plague is transmitted by flea bites, inhalation of
contaminants, or direct contact with infected animals (a). It is not prevented by vaccine (b); instead,
control of rats and the fleas that live on them is the best prevention. The plague has not been virtually
eradicated (c); it is still found everywhere, worldwide. Finally, the good news is that despite its
reputation, the plague is not always fatal, as in (d). Early antibiotic therapy is very effective; however,
without treatment, mortality may occur quickly, in as early as in 5 days.
20. (Answer: d). Infectious organisms may invade outside their normally preferred host species if an
individual host is sufficiently immunocompromised (d)contrary to (a)but not always, as in (c).
Finally, choice (b) is also false; if an individuals immune system is strong, it can fight off invasion
by many types of pathogens; thus, even if an individuals immune system is compromised, that
does not necessarily mean a pathogen will invade.

CHAPTER 19 PHARMACOLOGY
1. (Answer: a). Veterinary technicians may be responsible for any aspect of medication administration
EXCEPT prescribing medication (a). Only licensed veterinarians may prescribe, diagnose, and
perform surgery. In the hospital ward, the veterinary technician is often the person calculating the
amount of drug to give (c), preparing the dosage, administering the drug (b), and observing its effects
(d).
2. (Answer: b). Extra-label drug use (ELDU), also called off-label use, means using a drug for another
purpose besides that indicated (b), and outside standard prescribing (d). This is not illegal (c) within a
valid veterinarian-client-patient relationship, except in the treatment of food-producing animals. Drug
compounding (mixing or making medications) (a) is often acceptable under the same conditions.
3. (Answer: d). The right concentration (d) is not listed in the five rights of drug administration. Instead,
the five rights of drug administration include right patient (a), right drug, right dose (b), right time (c),
and right route of administration. The correct dose of a medication could be measured from different
concentrations of that medication, so concentration (d) is not a constant. For example, a dose of 100
mg can be measured from a 100 mg/mL concentration (1 mL) and a 50 mg/mL concentration (2 mL).
4. (Answer: c). The three checks technique is used when filling prescriptions or medication orders to
ensure that the correct medication is chosen (c). Look at the label when selecting the container; read
the label while removing the medication from the container; look at the label again when returning
the container to storage. The other aspects listed here are addressed in the five rights of correct
medication administration (a, b, and d).
5. (Answer: a). Although some nematodes (a) are very small, they are not as a group categorized as
microscopic life. Antimicrobial drugs are effective only against microbes such as viruses, fungi, and
bacteria (b, c, and d).
6. (Answer: b). The full course of antimicrobial medication must be given (b) to ensure that the infection
is cleared. Symptoms of infection may improve or disappear before all of the pathogens are killed, so
symptomatic treatment is not appropriate (a). Some medications will be better tolerated or absorbed
from a full stomach (d), but this is not consistent: others require an empty stomach or a stomach free
of certain foods (such as dairy).
7. (Answer: a). Sulfa drugs disrupt microbial metabolic activity (b). Other antibacterial drugs may work
by any of several mechanisms to prevent DNA synthesis (b), interfere with cell wall formation (c), and
interfere with the microbial protein synthesis (d).
8. (Answer: a). Opioids block the pain impulse in the brain (a) and therefore reduce the perception of
pain, but they do not prevent pain signals from reaching the CNS (b) or block perception of other
stimuli. Opioids do not have an antiinflammatory (c) or anesthetic (d) effect.
9. (Answer: d). In general, corticosteroids should not be given concurrently with NSAIDs such
meloxicam (d) because of the risk for additive adverse effects. There are no specific interactions of
concern between prednisone and midazolam, diphenhydramine, or famotidine (a, b, and c).
10. (Answer: d). Doxapram (d) stimulates the respiratory center in the brain. Dopamine and dobutamine
(a and b) are adrenergics, and dexmedetomidine (c) is an alpha-2 agonist.
11. (Answer: a). In animals, problem behaviors that can be observed (a) can be addressed. Internal states
such as bipolar disorder (b) and attention deficit hyperactivity disorder (d) would have to manifest in
particular behaviors to be recognized and addressed with medications and behavior modification
training. Chronic pain (c) is generally treated with analgesic medications and other pain-relieving
modalities.
12. (Answer: a). Antiarrhythmics are given to patients with cardiac arrhythmia in an attempt to
reestablish a normal pattern of electrical conduction within the heart muscle (a). They do not act on
nerve impulses in the brain, intestines, or skeletal muscles (b, c, and d).
13. (Answer: b). Epinephrine (b) is an adrenergic drug with direct stimulatory effect on the heart,
increasing rate and contractility. Atropine (a) is an anticholinergic drug with an indirect effect on heart
rate. Methylprednisolone (c) is a steroidal antiinflammatory drug. Sodium bicarbonate is an
alkalinizing agent (d).
14. (Answer: b). Chlorpromazine (b) is a phenothiazine drug with antiemetic properties. Ranitidine (a) is
a histamine-2 receptor antagonist used to reduce gastric acidity. Morphine (c) is an opioid
analgesicit can actually cause nausea and vomiting. Amlodipine (d) is a calcium channel blocker
used to treat hypertension.
15. (Answer: d). Narcotics (opiates) inhibit or reduce GI motility (d). Hyperosmotics and stimulants (b
and c) are used to treat constipation and can cause diarrhea. GI side effects of NSAIDs (a) can include
anorexia, vomiting, ulceration, and diarrhea.
16. (Answer: d). Hypothyroid patients usually require supplementation with levothyroxine (T ) (d) and
4

sometimes liothyronine (T ). Methimazole, radioactive iodine, and thyroidectomy (a, b, and c) are
3

treatments for hyperthyroidism.


17. (Answer: b). Azathioprine is used to suppress the immune system (b) in cases of autoimmune disease.
Antitoxins contain antibodies to toxins (c); antisera contain antibodies to pathogens (d);
immunostimulants are used to boost immune system function (a).
18. (Answer: c). Chemotherapeutics are toxic to dividing cells (c) and have their most significant effects in
tissues with rapidly dividing cells, not only in tumors (b), but also in bone marrow and the GI tract.
They are useful in humans as well as animals (a). Cytotoxic drugs should be handled with extreme
caution (contrary to choice [d]) to avoid inadvertent personnel exposure.
19. (Answer: a). Nutraceuticals fall into a gray area; they are not regulated as foods (d) or drugs (b), but
they are generally considered to be foods (or substances derived from foods) with therapeutic
(druglike) effects (c). Nutritional supplements (a) are added to the diet to ensure that nutrient
requirements are met.
20. (Answer: c). Prebiotics provide food for gut flora, and probiotics contain beneficial bacteria that can
help to restore normal gut flora, so they share the role of supporting healthy gut flora (c). Other effects
are more divided. For instance, lactoferrin and milk thistle may have antiinflammatory effects (a);
chondroprotectives may help rebuild joint tissue (b) and fluid; echinacea and L-lysine may be helpful
adjuncts to specific antimicrobial therapy in certain infections (d).

CHAPTER 20 PHARMACEUTICAL CALCULATIONS


1. (Answer: a). You need 25 mL sterile water to make a 4% solution using 1 g of drug.

2. (Answer: a). It is accurate that 6 mm = 0.006 m (choice [a]). Calculate: Move the decimal place to
the left 3 spaces.
3. (Answer: c). Take 1000 mL of 45% solution and add 2 L of sterile diluent (c).

4. (Answer: d). All three responses are appropriate (d). There must be a veterinarian-client relationship
to dispense any type of prescription drugs (a and b). The veterinarian must examine the patient before
dispensing any drugs (b). Finally, the veterinary technician cannot prescribe drugs, and these require
a prescription (a and c).
5. (Answer: a). The approximate drip rate is 10.3/10 seconds (choice [a]).

6. (Answer: d). To make 500 mL of 50% solution, take 250 mL of pure solution and add 250 mL of sterile
water.
7. (Answer: a). The final concentration is 25 mg/mL with a percentage of 2.5%.

8. (Answer: a). If 30 mL of solution is added to 70 mL of water, the final solution (v/v) is 30%.

9. (Answer: c). Give 2 tablets for 1 dose, and 4 tablets are needed over a 24-hour period. 20 kg 5
mg/kg = 100 mg. The dose is twice per day for 10 days. The tablet size is 50 mg therefore 100 mg/50
mg = 2 tablets per dose and 4 per 24-hour period
10. (Answer: c). To prepare 1.5 L of a 1:200 (w/v) solution given a 12% solution and sterile water, take 62.5
mL of the 12% solution and add 1437.5 mL of sterile water.

11. (Answer: a). Premix 1 mL/10 kg = x/26 kg = 2.6 mL. Propofol dose range 1 mL /4 kg = x
mL/26 kg = 6.5 mL and 1 mL/5 kg x mL/26 kg = 5.4 mL. Atropine dose range (26 kg
0.02)/0.5 = 1.04 mL and 26 kg 0.04 / 0.5 = 2.08 mL.
12. (Answer: b). (0.3 10 )/100 = 3000 ppm.
6

13. (Answer: c). Calculate: 5 g/100 mL = 5000 mg/ 100 mL = 50 mg/mL


14. (Answer: a). Calculate: 27.5 kg 2.204 = 60.5 lb
15. (Answer: d). Calculate: 25 mg/mL = x/0.3 mL = 0.3 25 = 7.5 mg
16. (Answer: b). Take 2 tablets (tabs) 4 times a day (q.i.d.) for 2 days (d), then 1 tablet (tab) once a day
(s.i.d.) for 10 days or as required (prn) before meals ([a] and [c]). Recheck May 8. Dispensing labels
should give clear instructions to the owner. The number of tablets, times per day, and recheck
statements should be included on the label.
17. (Answer: b). Add 215 g/hr or 45.2 mL fentanyl to 1000 mL of 0.9% sodium chloride.

45.2 mL of fentanyl should be added to 1 L of sodium chloride


18. (Answer: a). Calculate: 550 mL + 500 mL = 1050 mL.

19. (Answer: a). Give 3 drops (gtt) in left eye (OS) every (q) 4 hours for 3 days. Then as needed (prn). No
Repeats (NR). Prescription labels should be very specific, with no ambiguity. The name of the patient
should be noted, as well as how much of the drug and how to administer it. The length of the
administration of the drug should also be noted on the label.
20. (Answer: d). Convert 80 mg to grams = 0.080 g

For a vial that contains 80 mg of drug in 2 mL of injection, 0.5 mL of the injection should be
administered to obtain 0.02 g of drug.

CHAPTER 21 ANESTHESIA
1. (Answer: c). Preanesthetic drugs are not used to stimulate respiratory rate (c). Instead, they provide
mild sedation (b) and analgesia (a) for preoperative, intraoperative, and postoperative phases and
reduce anxiety (d), calming the patient.
2. (Answer: c). Neuroleptanalgesic (a) is a combination of a tranquilizer and/or sedative with an opioid
(analgesia). Anticholinergics (d) are parasympatholytic drugs that exert their effects by blocking the
actions of the parasympathetic neurotransmitter acetylcholine at the muscarinic receptors.
Cyclohexamine (a) is a specific classification of anesthetic called dissociative drugs that interrupt
neural transmission that induce unconscious and conscious brain functions. Barbiturates (b) are based
on the chemical substitutions on the barbituric acid molecule and the duration of action.
3. (Answer: b). Tranquilizers (b) can calm anxieties without altering consciousness. Anticholinergics (a)
are parasympatholytic drugs used to prevent or treat bradycardia. A sedative (c) is a tempting choice,
but it reduces excitement or irritability by causing decreased activity. Finally, isoflurane (d) is an
inhalant anesthetic agent used to produce unconsciousness and general anesthesia.
4. (Answer: d). Atipamezole is an alpha-2 antagonist that is specific to dexmedetomidine (d), which is
its opposite, an alpha-2 agonist. Ketamine (a) is a dissociative anesthetic drug; butorphanol (b) can
reverse the effects of morphine. Finally, acepromazine (c) is a tranquilizer that can be used in
combination with ketamine to reduce seizures in dogs.
5. (Answer: c). Pure mu agonists include morphine (a), meperidine (b), hydromorphone (d), and
fentanyl, which stimulate all opioid receptors. By contrast, butorphanol (c) is an agonist-antagonist
opioid.
6. (Answer: b). Only the 2-year-old St. Bernard with the hernia should be put on this drug (b).
Anticholinergics are contraindicated in patients undergoing upper GI barium series and endoscopy
(d); any patient with tachycardia (c) or ventricular arrhythmias; and geriatric patients with organ
dysfunction (a).
7. (Answer: c). Morphine and meperidine (a and b) have been reported to cause histamine release when
administered intravenously.
8. (Answer: b). Delirium or excitement occurs in stage 2 (b). Effects of each stage are as follows:
Stage 1: induction stage, stage of analgesia, and altered consciousness
Stage 2: stage of delirium or excitement, loss of consciousness
Stage 3: stage of surgical anesthesia
Stage 4: stage of medullary paralysis (apnea and cardiac arrest)
9. (Answer: c). Thiopental (c) is the only extremely alkalinic solution. If tissue sloughing accidentally
occurs, dilute the perivascular thiopental by injecting small amounts of normal saline around the site.
The other drugs are not associated with tissue necrosis. For example, dexmedetomines (a) side
effects include cardiovascular effects; pentobarbitals description (b) specifically notes that it can
be administered IM for sedation without tissue reaction. Finally, methohexital (d) is associated with
profound respiratory depression.
10. (Answer: a). Propofol (a) is a short-acting hypnotic and alkylphenol derivative. It is rapid acting with
smooth, excitement-free inductions, rapid smooth recovery as a result of the redistribution, and rapid
metabolism (redistribution to vessel-rich areas such as brain rather than muscle or fat). Diazepam (b)
is a member of the benzodiazepine family, which act as muscle relaxants and anticonvulsants, and are
classified as tranquilizers (mild calming effect). They are additives or synergistics that are used with
other drugs. Ketamine (c) is a cyclohexamine with selective analgesic properties, used in conjunction
with benzodiazepines for induction, but not used alone. Thiopental (d) is commonly used as an
induction agent, but it is not used for maintenance anesthesia because of the protein binding and lipid
solubility effects.
11. (Answer: b). The diameter-indexed safety system (DISS) (b) is a set of connectors that are specific to
each gas. The flow meter (c) measures and indicates flow of gases into the vaporizer and then to the
patient, and the check valve (a) located in the E tank yoke, anesthetic machine pipelines, and
regulators, ensures a one-way safe flow of gases from the regulator, tank, and/or pipeline. Finally, the
pressure relief valve (d) is also known as the pop-off valve and does just what its name implies. It is
located on the circle systems carbon dioxide absorber or Bain system mount.
12. (Answer: d). The oxygen flush valve can actually create increased pressure within the breathing
system when connected to the patient, not relieve pressure (d). In an oxygen flush valve, oxygen
bypasses the vaporizer (a), delivering 100% oxygen to the breathing system (b). Besides flushing the
system with pure oxygen, it also can fill the reservoir bag and system (c) to check for leaks.
13. (Answer: d). All three methods (d) are effective measures for reducing waste anesthetic gas exposure
in the environment. For example, directly attaching a scavenger hose (a) to an active scavenging
system will ensure there are no waste gas anesthetics going directly into the room or atmosphere.
Leak-testing an anesthetic machine (b) will identify any significant leaks, which could lead to waste
gas exposure. Finally, annual preventive maintenance (c) will ensure that the anesthetic machine is
functioning properly per the guidelines and recommendations set by the manufacturer. Preventive
maintenance will also ensure the safety of the patient and personnel.
14. (Answer: b). Halogenated anesthetic agents (i.e., isoflurane, enflurane) should not exceed 2 parts per
million (ppm) dose per day (b). It is acceptable to exceed 1 ppm (a), as long as 2 ppm is not exceeded,
which is recommended by Occupational Health and Safety Administration (OHSA). Finally, 3 ppm
and 4 ppm (c and d) would exceed OSHAs recommended daily exposure.
15. (Answer: b). In this list, only the analgesic (c) has the ability to significantly affect both vital
physiological signs and pathological effects. Antisialogogic drugs (d) prevent salivation. Antitussive
(a) agents suppress the cough reflex. Agonists (c) have an affinity for a specific receptor before they
produce an effect.
16. (Answer: c). NSAIDs. [c] Barbiturates, local anesthetics and Alpha2 agonists do not work peripherally
by reducing prostaglandins production on area of damaged tissue. Barbiturates (a) cause CNS
depression by acting on CNS neurons and GABA receptors. Local anesthetics (b) block sensory and
motor nerves and nerve transmission and Alpha2-agonists (d) stimulate alpha-adrenoreceptors
causing CNS depression and decreased catecholamine release.
17. (Answer: b). The only acceptable rate for ventilation during IVVP is 8 to 12 breaths per minute (b). All
other rates (a, c, and d) are incompatible with safe and ideal ventilation. The inspiration-to-expiration
ratio should be 1:2, with a tidal volume of 15 to 20 mL/kg (30 mL/kg for open chest), and an
inspiratory pressure of 12 to 20 cm H O (30 cm H O with open chest). The goal of using these
2 2

parameters is to decrease carbon dioxide levels to slightly below normal, thereby eliminating
spontaneous breathing and allowing control of ventilation.
18. (Answer: c). Capnography (c) is a graphic measurement of instantaneous carbon dioxide level of
respiratory gases and displayed as a waveform on a monitor. Electrocardiography (b) measures the
quality, rhythm, and rate of the heart. A Doppler (a) unit is a noninvasive blood pressure monitor that
also allows an audible heart rate sound and is especially useful for systolic pressure readings. Pulse
oximetry (d) measures the percent saturation of oxygen.
19. (Answer: c). A 35-kg dog requires at the minimum a 3-L reservoir bag and 10-mm or larger
endotracheal tube, eliminating choices (a) and (d). This could make it tempting to select choice (b).
However, the universal F circuit cannot be used with an endotracheal tube bigger than 9.5 mm
because the inner tube is not big enough to accommodate the larger endotracheal tube size. This may
result in hypoventilation. The correct choice, then, is (c).
20. (Answer: b). Respiratory acidosis is an acid-base disturbance in which the blood gas analysis reveals
carbon dioxide production is greater than carbon dioxide excretion (b). The increased carbon dioxide
causes a gain in acids and decrease in pH. By contrast, respiratory alkalosis is defined as carbon
dioxide excretion greater than carbon dioxide production (d), which causes excess loss of H ions (c)
+

and gain in bases (a).

CHAPTER 22 PAIN MANAGEMENT


1. (Answer: c). The veterinary technician can play an important role in pain management by
communicating directly with the clinician about particular concerns (c). Monitoring urine and fecal
output (a), although important, unless having direct bearing on a particular case, would not be the
best answer to contributing to pain management. Changing the medication when it is ineffective (b) is
important but cannot be done by a technician without input from the attending clinician. Directing
the veterinary assistant to give medications (d) is not the most helpful choice because it would be
important to see the animal receives the medication directly, not by passing the duty off to another.
2. (Answer: b). Three behavioral signs of pain in a cat include resentment at being handled, reclusive,
and abnormal posture (b). Sleeping continually, overeating, and attention-seeking behavior (a) are not
thought of as indicating pain. Combined, hyperactivity, pupillary enlargement, and tail swishing (c)
would not necessarily indicate pain. Finally, hypotension, hypocapnia, hypopnea, and bradycardia
(d) are not behavioral signs for pain.
3. (Answer: a). During hospitalization, a pain assessment should be conducted every 4 to 6 hours (a).
Doing this every 30 to 60 minutes (b) is not necessary unless this is the immediate postoperative
period, which is not specified in this question. Assessing pain less frequently, such as every 8 to 24
hours, is not enough for a hospitalized patient (c or d).
4. (Answer: b). To distinguish pain from dysphoria, try speaking in low tones and interacting with the
animal; if this makes the patient feel better but behaviors resume when interaction stops, this will
show that the patient has pain and is not dysphoric (b). Putting the patient on comfortable blankets
will not relieve an animal with dysphoria (a). Similarly, neither will moving the animal to a different
ward (c). Finally, automatically reversing the analgesic medication is not appropriate (d).
5. (Answer: c). AAHA/AAFP Pain Management Guidelines for Dogs and Cats (c) has elevated pain to
be the fourth vital sign. The other organizations are not listed in the chapter (a, b, and d).
6. (Answer: a). Nociception is the activity in the peripheral pathway that transmits and processes the
information about the stimulus to the brain (a). It is not a normal response to tissue damage (b), which
would be the definition for adaptive acute pain. Nociception is not pain without apparent biological
value that has gone on for some time (c); this is the definition of chronic maladaptive pain. The
definition for allodynia is pain from any stimuli to the affected area that would normally be
innocuous that becomes noxious (d).
7. (Answer: a). The correct sequence is as follows: Transduction, Transmission, Modulation, Perception
(a). The others are not in correct sequence (b, c, and d).
8. (Answer: d). Wind-up is an increase in the excitability of spinal neurons over time when a
painful stimulus is delivered repeatedly above a critical rate (d). This process is mediated in part by
the activation of NMDA receptors in dorsal horn neurons (a), but the two are not the same thing.
Wind-up is not a response to tissue inflammation (b); nor is it used to refer to any type of
physiological pain (c).
9. (Answer: c). In this particular list, only morphine (a) is an opioid agonist. Naloxone (b) is an
antagonist; and both butorphanol and buprenorphine (a and d) are partial agonists.
10. (Answer: d). Prostaglandins play an important role in mammalian renal physiology (d), where they
play a role in autoregulation of vascular tone, glomerular filtration rate (GFR), renin production, and
sodium and water balance. The other choices do not involve prostaglandinsproduction of
endorphins (a), production of leukotrienes (c), or mammalian cardiovascular physiology (b).
11. (Answer: a). Ketamine (a) is an NMDA antagonist medication. Lidocaine (b) is a sodium channel
blocker; meperidine (c) is an opioid; and dexmedetomidine (d) is an alpha-2 adrenoreceptor agonist.
12. (Answer: c). Gabapentin is an antiepilieptic agent. Sodium channel blockers (a) include drugs such as
lidocaine or bupivicaine); benzodiazepines (b) include drugs such as midazolam and diazepam.
Finally, nutraceuticals (d) include drugs such as glucosamine and chondroitin.
13. (Answer: b). (a) Buprenorphine, this is either injectable or transmucosal. (b) EMLA cream correct
answer. (c) Bupivacaine, this is injectable. (d) Fentanyl, this is either injectable or a transdermal patch.
14. (Answer: b). (a) Only used in large animal patients: this is not true. (b) Is an excellent block for canine
and feline orchiectomy (correct answer). (c) Requires the use of a vaporizer: not true, can be performed
on injectable anesthesia. (d) Never been performed in veterinary patients: not true, utilized routinely.
15. (Answer: c). All of these are benefits of CRI except choice (c). Although it is helpful that a variety of
drugs can be used in the CRI, drug choice should be based not only on what is best for the patient (e.g.,
analgesic potency and safety) but also on what is best for the hospital (e.g., comfort level with and
availability of drugs). The other advantages are trueCSI does enable lower doses and fewer side
effects (a), a more stable plane of analgesia (c), and greater control over drug administration (b).
16. (Answer: a). The three basic approaches to pain management in human physical therapy are manual
techniques, electrophysical agents, or physical modalities (c), and interventions that change movement
patterns thought to promote pain. Examples of manual therapies (a) include massage, as well as joint
mobilizations, stretches, therapeutic exercises, and soft tissue mobilization. Physical modalities (c) are
a different category. Effleurage (b) is a type of massage technique, and therapeutic exercise (d) is
another type of manual therapy.
17. (Answer: b). The beneficial effects of cryotherapy include vasoconstriction; reduced cellular
metabolism, decreased nerve conduction velocity, and decreased production of pain mediators,
leading to analgesia (a), as well as reduction of edema and decreased muscle spasm. Connecting
decreased metabolism so the patient is less hungry (a) is misleading and taken out of context; instead,
cryotherapy reduces cellular metabolism. Cryotherapy does not prevent hyperthermia (c) and is not
normally recommended for this condition. Finally, it is not true that it is most useful 2 to 3 weeks post
injury (d); instead, icing should be used for the first 48 hours.
18. (Answer: c). A myofascial trigger point is a pain phenomenon of soft-tissue origin characterized by a
hardened muscle band that is intensely painful on palpation (c). To call it a microlesion (b) is
misleading and inaccurate, and it is not a natural reflex (a). It is not a treatment strategy (d); the one
described in choice (d) is actually a description of acupuncture.
19. (Answer: d). Visceral pain is associated with internal viscera, such as that found in organs such as the
pancreas, stomach, and bowel (a, b, and c), and not within bone (d). Other causes of visceral pain
besides pancreatitis, gastroenteritis, and bowel ischemia include increased wall tension from
distention of the GI tract, biliary system, or urinary bladder and capsular distention of solid organs.
20. (Answer: b). (a) Lymphoma, diabetes, GDV (GDV is acute and this negates this answer). (b)
Osteoarthritis, IVDD, cancer (correct answer). (c) Respiratory infection, cardiac disease, blocked cat (all
3 are not chronic diseases). (d) Broken leg, bronchitis, hypertension (all 3 are not chronic diseases).

CHAPTER 23 SURGICAL PRINCIPLES


1. (Answer: c). Operating scissors (c) are best for cutting drape material. Metzenbaum (a) scissors have
fine tips and long handles for cutting and dissecting more delicate tissue. Littauer and Spencer suture
removal scissors (b and d) are used to cut and remove sutures postoperatively.
2. (Answer: a). Babcock intestinal forceps (a) have no gripping teeth, which enables them to be used on
delicate tissues. The Ferguson Angiotribe forceps (c) assist in holding large bundles of tissue, and the
Roeder (d) is a towel clamp; the Allis tissue forceps (b) have intermeshing teeth that ensure a secure
grip, but they can cause trauma to delicate tissue.
3. (Answer: d). Jones towel clamps (d) have a squeeze handle mechanism and are lighter weight and
more delicate than other towel clamps. Backhaus (a) towel clamps are considered forceps and are
used to secure drapes to the patients skin. Roeder towel forceps (b) are similar to the Backhaus
towel forceps, but have a metal bead on each tip that does not puncture the skin as deeply and helps
keep the drape from sliding. Finally, the Lorna (c) towel clamps are used to secure second-layer
drapes to the ground drapes.
4. (Answer: a). Rochester-Carmalt forceps (a) have longitudinal grooves and distal transverse grooves.
Rochester-Pean forceps (b) have transverse grooves; Rochester-Ochsner forceps (c) are similar to
Rochester-Pean forceps, but in addition have 1:2 teeth at the tips; and finally, Halsted (d) mosquito
forceps have transverse serrations along the entire jaw length. They are not, however, referred to as
Rochester-Halsted, but rather Halsted mosquito forceps.
5. (Answer: c). The Mayo-Hegar instrument (c) can be used only as a needle holder (no scissors). The
Halsted mosquito forceps (b) control capillary bleeders. The Doyen intestinal forceps (a) are useful for
holding bowel, and the Ferguson Angiotribe forceps (d) assist in holding large bundles of tissue.
6. (Answer: d). Start with this clue: the word part for bone is oste/o. The only instrument in this
list that cuts through bone is the osteotome (d). Wire-cutting scissors (a) are made to cut wire sutures.
Kern forceps (b) have strong gripping teeth; some have a ratchet to manipulate bone fractures to
reduction. Verbrugge forceps (c) can hold bone fragments in reduction while inserting fixators such as
screws.
7. (Answer: c). Taper-point needles (c) have reverse cutting points for tissues that may tear easily, such
as the bladder. Side-cutting needle points (a, b, d) are used for skin, cartilage, or tendons.
8. (Answer: b). Surgical gut (a) is made from sheep intestines; it is the only absorbable suture listed. The
otherssilk, cotton, and linen (a, c, and d)are not absorbable.
9. (Answer: d). Vetafil (d) is the only nonabsorbable synthetic suture listed. Polyglycolic, polyglactin,
and vicryl (a, b, and c) are all absorbable.
10. (Answer: d). Surgical hair removal should be completed with electric clippers and a no. 40 blade (d).
Thick-coated animals sometimes require clipping with the direction of hair first, perhaps using a no.
10 blade initially (a). No. 15 blades (b) refer to scalpel blades, not clipper blades, and are used for
precise, small or curved incisions. Likewise, a no. 25 blade (c) refers to a scalpel rather than clipper
blade. These are most commonly used for large animal procedures
11. (Answer: a). Chlorhexidine (a) is the antiseptic of choice for flushing the prepuce before surgery. Both
povidone-iodophor and isopropyl alcohol (b and d) are too tissue toxic. Roccal (c) is a disinfectant.
12. (Answer: c). Cleaning agents should have a pH between 9.2 and 11 (c) to prevent spotting and
corrosion. Anything lower (a or b) is not effective. A pH above that (d) is far too alkaline.
13. (Answer: c). Indicator tape does not give any indication as to what has happened inside the pack
(contrary to choice [c]); so choice (c) is the correct answer. Instead, the tape is used on the outside of
every pack to keep the seal tighter and prevent corners from unfolding. Lines on the pack will change
color to black (d) to indicate that the outside of the pack has been exposed to steam (b), but this does
not indicate appropriate temperature and time have been met (a).
14. (Answer: c). The outside of the pack is not sterile and should not be handled by the sterile assistant;
packs should be set out before scrubbing (c). More appropriate tasks include things such as keeping
the table and instruments organized (a), providing retraction (b), and cutting suture material (d).
15. (Answer: c). The Rongeurs is used to break up and remove bone; it would be used during an
orthopedic procedure (c). Onychectomy (a) is the removal of nails, celiotomy (b) is cutting into the
abdomen, and thoracotomy (d) means to cut into the trunk or thoraxbut that does not mean
breaking up or removing bone.
16. (Answer: d). Senn rake retractors (d) are double-ended handheld retractors useful for skin and
superficial muscle retraction; one end has a three-pronged point (sharp or blunt) that curves.
Meyerding, Hohmann, and U.S. Army retractors (a, b, and c) are handheld instruments for larger
muscle masses.
17. (Answer: b). A patient undergoing a gastrotomy is placed in a dorsal position (eliminating choices [a]
and [d]) and hair is removed from the xiphoid process to the pubis (eliminating choice [c]), followed
by disinfecting with chlorhexidine surgical scrub alternating with sterile saline and/or a final
preparation of 10% povidone/iodine paint solution (b).
18. (Answer: b). A no. 11 blade (b) is used to sever ligaments. A no. 10 blade (a) is the basic blade and is
commonly used for incising skin. A no. 12 blade (c) is used to lance abscesses, and a no. 15 blade (d) is
used for precise, small or curved incisions.
19. (Answer: d). Most lubricants are permeable to steam and will not affect sterilization, so they do not
have to be rinsed off (d). Surgical milk is an excellent lubricant (b) that inhibits rust (b) and should be
used on all instruments cleaned by ultrasound (c).
20. (Answer: c). Closed gloving (b) minimizes the chance of contamination. Open gloving (b) is a common
alternative. However, alternating (a) and semiclosed (d) are not identified as types of gloving
procedures.

CHAPTER 24 SMALL ANIMAL NURSING


1. (Answer: a). Sulfur and salicylic shampoos (a) would be hydrating for the skin and an appropriate
treatment for seborrhea sicca. Benzoyl peroxide (b, c, and d) is used to treat hot spots, skin fold
dermatitis, and deep pyodermas. Coal tar (b, c, and d) is used for allergic contact dermatitis; and
chlorhexidine (d) is used for hot spots and superficial folliculitis.
2. (Answer: c). A thymoma is (b) a tumor of the thymus (thym/o) that is always malignant; this is an
exception to the rule of oma, which normally indicates a benign tumor (d). It has nothing to do
with thyroid cartilage (a), or the hypothalamus (b).
3. (Answer: b). Referral sounds from the upper airway are normally coming from the trachea (b). These
are generally respiratory sounds such as wheezing, crackling, or coughing and therefore are from the
trachea, as opposed to the diaphragm, lung lobes, or digestive tract (a, c, and d).
4. (Answer: b). The Kbler-Ross five stages of grief usually occur in the following order: denial,
bargaining, anger, depression, and either resolution or acceptance (b). Denial tends to come first
because initially we usually cannot believe what we are hearing and we react with confusionthe
denial helps us test the truth until we can absorb it. Timing of absorbing disconcerting news is not
usually conducive to an immediate plunge into depression (c) before we can take in the news itself.
Bargaining (a and d) cannot come first either because, as before, we need to initially understand what
is happening.
5. (Answer: c). The parenteral route never includes oral medications (b). It does, however, include
injectable or topical routes (a, and d).
6. (Answer: c). If a patient is overhydrated, the heart rate and respiratory rate would increase (c)
because of the increased fluid volume in very vascular tissue. Decreased capillary refill, decreased
respiratory rate, and hypotension (a, b, and d) are associated with dehydration.
7. (Answer: c). Plasma (c) is the only fluid in this list considered a colloid; colloids are solutions
containing protein or starch molecules. The rest (a, b, and d) are crystalloids.
8. (Answer: b). The patient needs dextrose (b), a hypotonic crystalloid. Mild hydration, small patients
and evidence of chronic heart failure (a, c, and d) are all patients that could receive small amounts of
subcutaneous fluids.
9. (Answer: a). Abdominal radiographs are recommended to evaluate the patients abdominal
organs. The signs of constipation may also indicate neoplasms or a foreign body. Abdominal
palpation (b) may not detect a foreign body. Intravenous fluids (c) are not necessarily required
preenema. (d) Large amounts of laxative may not be required or not recommended for an obstructed
patient.
10. (Answer: b). 25 kg . Blood donors must be a large size dog to allow for collection of a large volume of
blood. (a), (c), (d), are too small of a donor to collect a large quantity of blood.
11. (Answer: a). Treatment of an abscess. (b) Drains would not be needed in a clean wound. (c) Removal
of necrotic tissue is called debridement. (d) The removal of air and plasma from the wound, drains are
used to remove air, exudate and serous fluid in necrotic or contaminated wounds.
12. (Answer: a). Non adherent primary, secondary and the outside layer or tertiary layer (a) is the correct
answer. Non adherent is a type of primary bandage. The secondary layer is on top of the primary layer
and the last layer is called tertiary. The tertiary layer is used to support the primary and secondary
bandages. (b, c, and d) are in the wrong order or incorrect terminology.
13. (Answer: d). (d) Increase intravascular volume. Hypotonic crystalloids such as 5% dextrose in water
(D W), 0.45% NaCl, Normosol M and Plasmalyte 56 are used to increase the intravascular volume.
5

(a) Colloids are used for edema only. (a) and (b) Colloids do not increase the protein in the blood nor
decrease interstitial blood volume.
14. (Answer: c). All these symptoms indicate moderate, or 8%, dehydration (c). Severe dehydration of 10%
or greater (d) would show more severe forms of these same symptoms, including altered state of
consciousness. The text does not discuss 3% dehydration (a), but 5% alone (b) is mild, involving
minimal loss of skin turgor and semidry mucous membranes.
15. (Answer: a). The amount of Normsol R replacement fluids for a 36.5-kg dog who is estimated to be 8%
dehydrated would be 2920 mL (a).

16. (Answer: a) PVCs can occur at any rate but pose a greater danger when occurring with a sustained
tachycardic heart rate (a). (b) PR interval represents the beginning of atrial depolarization into
ventricular depolarization. (c) Third degree heart block is also known as a complete heart block; the
most severe heart block. (d) The cells of the heart are electrically linked; therefore the depolarization
spreads quickly from the sinoatrial node to the atria in a caudal direction toward the ventricles, finally
reaching the atrioventricular (AV) node.
17. (Answer: b). Rose-Bengal stain can be used to stain devitalized tissue and to diagnose
keratoconjunctivitis sicca (KCS) (or dry eye) (b). The stain for corneal abrasions (a) is fluorescein stain.
There is no diagnostic test for vitamin deficiencies (c) other than serology testing. Finally,
conjunctivitis (d) is usually diagnosed after a complete examination of the eye.
18. (Answer: a). Gonioscopy can be used for the diagnosis of glaucoma by magnifying the lens to examine
the iris angle and anterior chamber (a). It is not used to examine either the esophagus (b) or trachea
(d), and it cannot help in the detection of KCS (c).
19. (Answer: c). Ringworm is caused by a fungus, such as Microsporum canis and Trichophyton spp (b).
Malassezia pachydermatis (c) is also a fungus, but it does not cause ringworm. Staphylococcus spp.,
Streptococcus spp., Pasteurella spp., and Pseudomonas spp. (a) are bacteria, not fungi; and Otodectes
cynotis (d) is a parasite.
20. (Answer: d). A wound is considered infected if the patient is presented for treatment more than 48
hours post injury. (a) A contaminated wound does elicit a response. (b). A surgical wound is
considered contaminated by suture material. (c) Not only microorganisms can contaminate a wound.
All wounds are contaminated; however, a contaminated wound elicits no immune response from the
host body.

CHAPTER 25 EQUINE NURSING AND SURGERY


1. (Answer: d). It is not true that long-term Mila catheter placement is meant to be used for regional
perfusions (d). Regional perfusions are performed to deliver antimicrobials directly into the site of an
infection such as into a joint or tendon sheath. For this, you need a butterfly catheter, not a Mila long-
term catheter. The rest are true. Mila catheters do use a guide-wire system, both hands must be kept
sterile, and it is commonly placed in the jugular of neonatal foals (a, b, and c).
2. (Answer: d). If the procedure is done correctly, the veterinary technician should be able to observe
the tube passing along the left jugular groove as it advanced down the esophagus (d). Eliciting the
gag response (c) is definitely not a goal of this procedure. Instead, sucking back or blowing to get
positive pressure (a), the technician should draw back on the tube with dosing syringe or mouth to
check for negative pressure because the esophagus will collapse around the end of the tube. Seeing
the tube on the right side of the neck (b) would be a big mistake because the tube should be passing
along the left jugular groove as it advances. The tube should never be seen on the right side of the
neck.
3. (Answer: a). Spores of Clostridium botulinum in soil or vegetation (c) can enter the wound causing
botulism. It is not transmitted via stepping on a nail, mosquito, or a wound abscess (a, b, and d).
4. (Answer: c).Lawsonia intracellularis infects the horse with Equine proliferative enteropathy (c) by the
ingestion of contaminated feces from domestic or wild animals. Lawsonia surs (a) is not correct. Signs
include lethargy, fever, weight loss, diarrhea, colic and anorexia, not pneumonia and gastrointestinal
tract abscesses in weanlings (b), so (d) is also not correct.
5. (Answer: c).Rhodococcus equi is a coccobacillus bacterium found in the soil. It causes respiratory
symptoms including pneumonia (c). It does not cause hypoproteinemia, diarrhea or chronic weight
loss (a, b and d).
6. (Answer: d). If the first premolars (P1) also known as wolf teeth (d) do not fall out on their own, a
veterinarian will have to extract the teeth because they can interfere with the bit. The others (a, b, and
c) are not normally extracted except in cases of significant pathological processes.
7. (Answer: b). Rasping down the sharp edges (b) on the buccal surface of the upper teeth and lingual
lower teeth is called floating the teeth. Anatomically, the horses upper jaw is wider than the
lower jaw. When they eat, they grind their food in a side to side motion. This motion creates sharp
edges on the buccal surface of the upper teeth, and on the lingual surface of the lower teeth.
8. (Answer: c). Equine herpesvirus 1 (EHV-1) primarily affects the reproductive system (c) and not the
other systems (a, b, and d). EHV-1 has been known to cause abortion storms among mares that are
housed together. It also causes the birth of weak and sickly foals that often die at or shortly after birth.
9. (Answer: c). Broken teeth (c) are not an indication for teeth to be floated. If, on the other hand, a horse
has halitosis, difficulty eating, or lacerations in the oral cavity, these may indicate too-rough edges
that need to be ground and floated (a, b, and d).
10. (Answer: d). When giving an IM injection in the neck, the proper injection site is defined by three
anatomic structures. Using these structures as a guide, they create a visual triangle: nuchal ligament at
the crest (top) of the neck, cervical vertebrae at the bottom of the neck, and the shoulder blade at the
base of the neck (d). The IM injection is given in the middle of this triangle, near the base of the neck.
The other locations (a, b, and c) are not the correct criteria for this injection.
11. (Answer: d). The bacterium Clostridium botulinum has NOT been implicated as one of the possible
causes (d). Instead, Cl. botulinum causes botulism, even though Clostridium perfringens and Clostridium
difficile are both possible causes of colitis, an acute inflammatory process (a) that alters gastric motility
(c); its causes can be speculated, but are largely unknown (b).
12. (Answer: b). Laminitis is inflammation of the sensitive laminae of the foot (b). This condition is very
painful, making it difficult for the horse to walk or stand comfortably. It is not an inflammation of the
carpus and fetlock joints (a), nor is it an abscess on the sole of the foot (d). Laminitis is not a spinal
cord inflammation (c).
13. (Answer: a). In the most extreme cases of laminitis, the coffin bone rotates distally (downward), and
can come through the sole of the foot (a). In extreme laminitis the horse does not want to move so will
not stall walk (c); instead, the condition causes continual, not intermittent (b), lameness. Degeneration
of the navicular bone (d) is called navicular syndrome, not laminitis, having nothing to do with the
laminae.
14. (Answer: c). Navicular syndrome is a degeneration of the navicular bone. The most common clinical
signs are frequent stumbling, a shortened stride, and intermittent lameness (c). Toe pointing, rocking
back on the heels, pyrexia, bounding digital pulses, and increased heat in the hoof wall (a, b, and d)
are all common clinical signs of laminitis.
15. (Answer: b). Vaccinations are routinely given by intramuscular route (b) with the exception of
strangles and influenza, which are given intranasally. Vaccinations are not routinely given
subcutaneously (c) nor are they given IV or ID (a and d).
16. (Answer: d). Equine infectious anemia (d) is a highly contagious and reportable disease. There is no
cure and no vaccine for it. There are routine vaccines for rabies, tetanus, and EEE/WEE (a, b, and c).
17. (Answer: a). Within 24 to 48 hours after vaccinations, some horses may show side effects of
generalized muscle pain and mild lethargy and have a mild fever (a). These are not unusual. Runny
eyes, nasal discharge, and colic (b) are not typically seen, whereas laminitis and head shaking,
sweating, and anxiety (c and d) would indicate something serious but are not typically associated with
vaccination.
18. (Answer: c). HYPP is not curable (c). Instead the aim of treatment is to reduce clinical symptoms by
administering acetazolamide 2 to 4 mg/kg orally, every 8 to 12 hours, hydrochlorthiazide 0.5 to 1
mg/kg orally, every 12 hours, and feeding a diet that is low in potassium, not high in it. The features of
the disease described in choices (a), (b), and (d) are all true.
19. (Answer: b). Potomac horse fever is caused by Neorickettsia risticii, and the vector is aquatic insects and
fresh water snails (b). Rabies is caused by Lyssavirus (a), and EPM is caused by a protozoa found in
opossum feces (c). Rhodococcus equi is caused by the bacterium Coccobacillus found in the soil (d).
20. (Answer: a). The equine influenza A (flu) virus can be transmissible to another animal by coming into
contact with infected fomites and through aerosolization, including through secretions from a cough
(a). It can survive for hours in the environment. It is not caused by ingestion of contaminated feces,
spoiled food, whole blood (b), open wounds, umbilicus, or surgical incisions (c) or by aquatic insects,
birds, or mosquitoes (d).
CHAPTER 26 LARGE ANIMAL NURSING, SURGERY, AND
ANESTHESIA
1. (Answer: d). Jersey (d) is the only dairy breed. Charolais, Hereford, and Angus (a, b, and c) are all
cattle beef breeds.
2. (Answer: b). The Boer goat (b) is used primarily for meat production. Alpine, LaMancha, and
Toggenburg (a, c, and d) are all goat dairy breeds.
3. (Answer: a). The size of the syringe is not important to the selection of needle size. The rest are (b, c,
and d).
4. (Answer: a). Performing an IM in the hind quarters should not be completed because of possible
damage to favorable cuts of meat. The cervical muscles can be easily discarded and are not considered
economically favorable (a). The others are not true reasons (b, c, d).
5. (Answer: c). (a), (b) and (d) provide the routes to administer the largest volumes of fluids of the 4
routes provided. The subcutaneous (c) route does not allow large volumes to be administered.
6. (Answer: a). If individual testing is to be completed such as for the California Mastitis test, each teat
should be cleaned in the order of far to near then sample from the nearest side first to decrease the
occurrence of cross contamination (a). The others are incorrect (b, c, and d).
7. (Answer: a). Intravenous calcium should be given slowly to prevent cardiac arrest (a). Rapid
administration of a cardiotoxic drug is counterintuitive (b). Calcium is not irritating to vascular
tissues (b). IV administration of sterile water is hypotonic and can cause hypernatremia (d).
8. (Answer: a). Urinary catheterization of bulls is difficult to impossible because of the anatomic
structure of the sigmoid flexure and the urethral diverticulum (a). The urethral process is the problem
in catheterizing rams, bucks, and machos, not bulls (b and c). Bulls do not have an os penis (d).
9. (Answer: d) Free gas is treated by relieving the pressure in the rumen (d). Frothy bloat is treated with
surfactant (a). Increase of carbohydrates exacerbates bloat (b). Decrease of water will increase
dehydration (c).
10. (Answer: c). Magnets are used to prevent hardware disease by attracting miscellaneous metal
consumed by cattle.
11. (Answer: b). Q fever, Bangs disease and Listeriosis (b) are all zoonotic and abortive (b).
Trichomoniasis, and Mucosal Disease are not zoonotic (d) and abortion is not a common sequela of
Rabies, ORF, or Mucosal Disease (a, c, and d). Although neosporosis (c) can cause abortion, it is not a
proved zoonotic at this time.
12. (Answer: a). BVDV (a) is the only virus on the list that can result in a persistently infected animal. The
others do not (b, c, and d).
13. (Answer: c). Bluetongue (c) is spread through contact with an insect known as a midge. Listeriosis (b)
is caused by gram-positive, nonsporing coccobacilli. Anaplasmosis (d) is a rickettsial infestation that
can be transmitted by arthropod vectorsmainly ticks, but not midges. Trichomoniasis (a) is a
protozoal infection.
14. (Answer: a). Contagious foot rot in sheep is caused by anaerobic bacteria (a) Bacteroides nodosus that
acts synergistically with Fusobacterium necrophorum. It is not fungal, protozoal, or viral (b, c, and d).
15. (Answer: b). When a large animal is in lateral recumbency during anesthesia, the lower forelimb needs
to be pulled forward (b); this will help prevent radial nerve paralysis. None of the other positions (a, c,
or d) will prevent radial nerve paralysis in this patient.
16. (Answer: c). Regional anesthesia used for laparotomies (c) requires anesthesia of a large area, but
innervation is from the spinal cord in a cranial-to-caudal, dorsal-to-ventral direction requiring a line
block or infiltration of direct nerve roots, not a ring block. Ring blocks are frequently used, however, in
places requiring smaller areas of anesthesia. In such cases, injection of a local anesthetic in a circle
around the circumference of a teat or a lower limb can be used for such procedures as dehorning, teat
repair, and claw amputation (a, b, and d).
17. (Answer: d). All the answers are correct (d). Feed or water in the rumen causes pressure on the
diaphragm, decreasing pleural space, resulting in decreased respiratory capacity or ventilation
compromise (c) and cardiac space, as well as relaxation of the cardia. This can cause bloat (a) as
fermentation continues in a full rumen. Anesthetics cause relaxation of the cardia, allowing for
regurgitation (b) of rumen contents.
18. (Answer: b). Elastrator bands (b) are the only instrument listed that would provide a bloodless
castration or tail docking. The Newberry knife and emasculator (d and c) do not provide bloodless
results, and a branding iron (a) is not used for castration or tail docking.
19. (Answer: d). Epidural anesthesia (d) is used to block the perineal region and reduces the urge to
strain. The other answers (a, b, and c) would not be effective.
20. (Answer: b).Clostridium novyi (b) causes Black disease, or type B infectious necrotic hepatitis.
Clostridium sordellii (c) causes gas gangrene (malignant edema), Clostridium chauvoei (a) causes blackleg
(clostridial myositis), and, Clostridium haemolyticum (d) causes bacillary hemoglobinuria.

CHAPTER 27 EMERGENCY MEDICINE


1. (Answer: c). A patient with nonhemorrhagic vomiting and diarrhea is an urgent patient and should
receive care or be reevaluated within 15 to 45 minutes (c). A resuscitative or nonstable patient should
receive care immediately (a) for life-threatening conditions. An emergent patient, such as a patient
with a serious laceration or fracture, should receive care or be reevaluated within 5 to 15 minutes (b).
Finally, a stable patient is not exhibiting a life-threatening condition. These patients should receive
care or be reevaluated within 1 to 2 hours (d).
2. (Answer: d). Respiratory rate, depth and effort (correct answer). (a) Respiratory rate (needs effort and
depth too). (b) Respiratory effort (needs rate and depth too). (c) Respiratory depth and rate (needs
effort too).
3. (Answer: d). Mucous membrane color, CRT, heart rate, pulse rate and pulse quality all need to be
accessed and provide important information.
4. (Answer: b). Chest compressions, ventilation, drugs, defribrillation and open chest compressions are
advanced life support treatments. The other answers (a),(c), (d) are not complete answers.
5. (Answer: b). The abbreviation CPCR stands for cardiopulmonary cerebral resuscitation (b). The other
options suggested in this question are made up (a, c, and d).
6. (Answer: c). Large, round, barrel-chested dogs can be in either right lateral or dorsal recumbency (c)
with hands directly over the heart. Other animals are placed in right lateral recumbency (a). (b) Left
lateral is ineffective in large dogs. Sternal recumbency (d) would make chest compressions ineffective.
7. (Answer: a). In an emergency situation, when venous access cannot be obtained immediately, the
needed drug should be administered via endotracheal tube at twice the IV dose (a). The same dose as
IV (b) or less (b or c) will not be potent enough by this route because of distribution or delivery
method. Intracardiac is not recommended (d).
8. (Answer: d). The pulse oximetry (SaO ) reading provides the percentage of available hemoglobin that
2

is saturated with oxygen (d). It does not identify the amount of total hemoglobin (b), but the
percentage available that is saturated; nor does it tell you the amount of oxygen (a), but the
percentage of hemoglobin that is saturated with oxygen. Pulse oximetry is not a measure of carbon
dioxide expiration (c).
9. (Answer: b). Cardiogenic shock (b) is poor tissue perfusion as a result of decreased forward blood
flow from the heart. This can be confused with hypovolemic shock (a), in which the poor tissue
perfusion is a result of low blood volume, frequently seen as a result of trauma. Septic shock (d) is
caused by the presence of toxins in the blood or other tissues; and anaphylactic shock (c) is is caused
by a hypersensitivity to an allergen.
10. (Answer: d). 100-150 ml/kg/min (correct answer). (a) 2-3 L/min (flow rate for flow by oxygen). (b) 2-5
L/min (flow rate for oxygen hood, mask or cage). (c) 40 ml/kg/min (flow rate for oxygen delivery via
an endotracheal tube).
11. (Answer: c). In head trauma, place the patient on flat board and elevate the board to approximately 30
degrees (c) in an attempt to decrease intracranial pressure. A 5% to 10% elevation is not enough (a and
b), and 75% is too high (d).
12. (Answer: a). Hypercalcemia is a medical emergency because of its effects on the kidney (a). The heart
(d) may seem like a strong choice, except that it is more affected by calcium administration, which can
cause arrhythmias or bradycardia. Hypercalcemia has no known effects on the spleen or liver (b or c).
13. (Answer: d). Gastric dilation (a) is characterized by abdominal distention; nonproductive retching;
and pale, muddy, or gray mucous membrane. The nonproductive retching and gray mucous
membrane are the biggest tip-offs herethey are not associated with intussusception, GI obstruction,
or peritonitis (a, b, or c).
14. (Answer: c). Calcium gluconate (c) is helpful in treating eclampsia. Atropine (b) is used to increase
heart rate, among other things; epinephrine (c) is used to increase heart rate; and dexamethasone (a) is
a steroid used to decrease inflammation, among other things.
15. (Answer: a). To treat hypoglycemia, supplement diet with a dextrose bolus diluted (a) 1:4 with a
crystalloid. This will help prevent phlebitis. The bolus should not be given undiluted (b). The SQ route
(c) is irritating to the skin because dextrose is hypertonic; the IM route (d) is equally undesirable
because the hypertonicity irritates the muscle as well.
16. (Answer: c). 2 hours is the correct amount of time between fetuses. hour and 1 hour is too short a
period of time (a and b) and (d) 4 hours is too long a period of time.
17. (Answer: d). Clinical signs of acetaminophen toxicosis show up 1 to 2 hours after ingestion and
include brown or cyanotic mucous membranes (d). Membranes that are pale (a) are associated with
hypovolemia; yellow mucous membranes (b), with liver conditions; and brick red (c) mucous
membranes, with septic shock.
18. (Answer: a). Normal values of urine output should be 1 to 2 mL/kg/hr. l to 2 mL/hr and 2 to 3 mL/hr
would both be too little (b and c). 2 to 3 mL/kg/hr (d) is more than required.
19. (Answer: c). An animal that is choking (patient 3) should be seen first (making choice [c] the only
correct answer). Establishing a clear airway and adequate respiration is always the first priority.
Second, the dehydrated animal (patient 2), especially an older one, should be seen second; it
represents an urgent situation, but not as urgent as choking. Third, respond to the animal with a fish
hook in its lip (patient 4), which has the potential to cause a further injury. All three should be seen
before a stable patient with a small laceration (patient 1).
20. (Answer: c). Respiratory distress (patient 3) should always be tended first. Dehydration and 3 days of
vomiting and bloody diarrhea (patient 2) should be seen next, especially in so small an animal. The
animal just hit by a car (patient 1) should be seen next. Pink mucous membrane and no obvious
distress means the patient can wait while the two more emergent cases of respiratory distress and
severe fluid losses are seen first. However, because patient 1s injuries from the car accident are
still fresh, the patient needs to be assessed for as-yet-undetected internal injuries, which may be
serious. The small, visible laceration (patient 4) can, in this case, go last.

CHAPTER 28 VETERINARY DENTISTRY


1. (Answer: a). In cats, the normal depth of the gingival sulcus is 0.5 to 1 mm (a). In dogs it is 1 to 3 mm
(c). The other two values (b and d) are incorrect.
2. (Answer: d). Squamous cell carcinoma (d) is the most common oral malignant tumor in cats and the
second most common in dogs. Melanoma (a) is most common in dogs, is rare in cats, and has a poor
prognosis. Fibrosarcoma (c) is the third most common in dogs and the second most common in cats.
Osteosarcoma (b) is not as common as the others.
3. (Answer: d). Mandibular molars (d) are the only ones listed here that are radiographed using the
standard parallel technique. The other teeth listed (a, b, and c) are imaged using the bisecting angle.
4. (Answer: c). The number assigned, 208 (c), is based on the modified Triadan system whereby 2
represents the left maxilla and 08 is the 8th tooth in the upper left quadrant, which is the fourth
premolar. Molars in dogs and cats begin with 09. The number 108 (a) is the upper right 4th premolar;
204 (b), the upper left canine. Tooth 308 (d) is the lower left 4th premolar.
5. (Answer: c). Sickle scalers remove supragingival calculi (c). That is, they remove calculi above the
gum line. They do not have just one sharp side (a) but rather have two cutting edges that taper to a
point. For removal of calculi below the gum line (b), curettes are more useful. Sickle scalers must
always pull away from the gum line, not toward it (d).
6. (Answer: a). The periodontal ligament holds the tooth in the alveolus (socket) (a) by attaching the
tooth at the cementum to the alveolar bone. By anatomic geography, then, the other connections (b, c,
and d) are not possible.
7. (Answer: a). Studies have indicated that excess vitamin D, or hypervitaminosis D (a), causes changes
similar to tooth resorptive lesions. Many cat foods contain excess vitamin D. Low pH in the diet (c) is
not associated with resorptive lesions. Evidence has not linked resorptive lesions to
hyperparathyroidism or calicivirus (b or d).
8. (Answer: a). Type 1 (a) lesions show a radiolucency on the tooth but with a normal periodontal
ligament space. Type 2 (b) show some radiolucency in the tooth and narrowing or loss (ankylosis) of
at least some of the periodontal ligament space. Type 3 (c) shows features of both type 1 and 2 on the
same tooth. Type 5 grading (d) does not exist.
9. (Answer: b). For scalers and universal curettes, align the terminal shank with the 90-degree line (b)
and align the stone with either of the 110-degree lines depending on the edge that is being sharpened.
The 110-degree line (a) opposite to the side to be sharpened is used for the Gracey curette. Thirty- and
0-degree angles (c and d) are not used in sharpening a universal curette.
10. (Answer: c). Anything within 2 meters, or 6 feet (c), of a scaler will be contaminated by scatter from
the instrument; 4 and 5 feet (a and b), of course, are within that distance and do not represent its full
reach; 8 feet (d) is beyond the estimated reach of bacterial contamination.
11. (Answer: d). The term buccal (d) means the surface toward the cheek; lingual (a) is the surface toward
the tongue; coronal (b) means toward the crown of the tooth; and palatal (c) refers to the surface
toward the soft palate.
12. (Answer: a). The upper fourth premolars overlap the lower first molars, forming the carnassial (a)
teeth in dogs and cats. These are the largest cutting teeth in these species. Dilacerated (b) is used to
describe an irregular or sharply angled surface. Diphyodont (c) means two sets of teeth, and, finally,
deciduous (d) teeth are primary teeth.
13. (Answer: b). Rotary scalers (b) spin at 300,000 to 400,000 rpm and can easily damage the enamel,
dentin, and soft tissue, and should be avoided in dental scaling. Sonic, ultrasonic, and piezoelectric (a,
c, and d) scalers are common mechanical scalers used in veterinary dentistry.
14. (Answer: a). Only the American Veterinary Dental College (AVDC) (a) has a standardized reference.
CVMA (b) stands for Canadian Veterinary Medical Association. AVMA (c) stands for American
Veterinary Medical Association; and NAVTA (d) stands for National Association for Veterinary
Technicians in America.
15. (Answer: a). Elongation (a) occurs when the x-ray beam is more perpendicular to the tooth, making
the tooth look much longer. Foreshortening (b) occurs when the x-ray beam is more perpendicular to
the film, not the tooth. A grayish overlay (c) results when light-sensitive CR plates are left out in room
light. Superimposition of mesial roots (d) occurs when using a lateral view with a bisecting angle
technique.
16. (Answer: c). For incisors and canine teeth, stand at the patients side (c) to appreciate how they
curve into the skull. For premolars and molars, stand at the patients front (a) to view how they are
seated in the skull. No image views covered in this text require the technician to stand at the rear or
ventral aspect to accurately position a patient (b and d).
17. (Answer: a). A rostral crossbite is an example of a class I malocclusion (a), in which maxillary and
mandible are correctly proportioned, but one or more teeth are misaligned. A class II malocclusion (b),
is referred to as distoclusion; teeth in the maxilla occlude rostral to mandibular equivalents. Referred
to as brachygnathism, overshot jaw, and overbite. Class III (c) is referred to as mesicoclusion: the
mandibular teeth occlude rostral to maxillary equivalents (and referred to as prognathism, underbite
or undershot jaw). Wry bite (d) is a nonspecific term that refers to a variety of unilateral occlusal
abnormalities where one segment of the jaw is disproportionately sized relative to the half. It is
generally not recommended to use this term.
18. (Answer: b). Class II malocclusion (b), also called distoclusion, in which teeth in the maxilla occlude
rostral to mandibular equivalents (overbite), is most likely to occur in dolichocephalic breeds with a
narrow skull and long maxilla, such as a collie. In class I malocclusion (a), the maxillary and mandible
are correctly proportioned, but one or more teeth are misaligned. In class III (c), the mandibular teeth
occlude rostral to maxillary equivalents (e.g., prognathism, underbite or undershot jaw). Wry bite (d)
is a nonspecific term that refers to a variety of unilateral occlusal abnormalities.
19. (Answer: b). Enamel defects are evident in permanent dentition if illness or injury occurred between 2
weeks and 3 months of age (b) during enamel formation (e.g., distemper). Enamel defects are also
evident in deciduous dentition if illness or injury occurred between day 42 of gestation until 15 days
postpartum (a, d) permanent teeth. This chapter does not suggest any permanent enamel dysplasia
resulting from a disease occurring at 5 months of age (c).
20. (Answer: c). PD3 (c) represents a 25% to 50% attachment loss. PD1 (a) indicates gingivitis only with no
attachment loss; PD2 (b), less than 25% attachment loss; and PD4 (d), greater than 50% attachment
loss.

CHAPTER 29 PERSONAL, PRACTICE, AND PROFESSIONAL


MANAGEMENT, SKILLS AND ETHICS
1. (Answer: c). Roughly 55% of communication is body language, 38% is tone of voice, and 7% is the
actual words spoken (c). People remember 10% of what they hear, 20% of what they read, but 80% of
what they see and do (a). The other combinations of percentages are random.
2. (Answer: b). Sales point displays (b) are special product displays that corporations design for use in a
clinics waiting areathis is internal, not external, marketing; it is focused on marketing to those
clients who have made it at least as far as the waiting room. External marketing activities are aimed at
expanding current client activity while also increasing the exposure of goods and services to those
who are currently not clients. Examples include website or internet marketing, marketing through
emails, and running a professional blog in which the practice addresses pertinent questions
consumers often have (a, c, and d).
3. (Answer: b). Communication barriers (b) include stereotyping and misread body language. In
downward communication (a), information travels from figures of authority to subordinates, relaying
instructions for the task at hand. Horizontal communication (c) is between people at the same level in
an organization. In upward communication (d), information travels from subordinates to authority
figures.
4. (Answer: c). Websites and social media are a great tool, but its very important to protect your
business from employees posting inappropriate or negative content (c). That said, creating an online
community bonds with clients outside the facility and is a great tool for managing the practice brand
and reputation, as well as staying connected with pet owners and delivering relevant pet care
information (a, b, and d).
5. (Answer: d). Use words that normalize problems and reduce resistance, such as, "in some situations,
"it is not uncommon for . . ." and "for some people in similar situations" (d). Words that are critical,
blaming, judgmental or accusatory (a, b, or c) tend to create a resistant and defensive mindset that is
not conducive to productive problem solving.
6. (Answer: d). Open questions encourage a more detailed response, self-expression, and involvement
in the conversation by preventing the opportunity to simply answer yes or nosuch
as asking the client to describe a symptom (d); this cannot be answered yes or no.
Questions (a), (b), and (c) can all be answered yes or no.
7. (Answer: d). We must have clear priorities and goals for what we want to achievethat is, balance
(d), then find the time to approach each with a sense of proportion within that balance. Although
efficiency, effectiveness, and prioritization (a, b, and c) are important concepts when dealing with
time management, the key to becoming successful is fulfilled by finding balance in life.
8. (Answer: c). The key here is in understanding the difference between a physical symptom and a
behavioral one. Overeating (c) is classified as a behavior that also may be exhibited by individuals
experiencing burnout, but it is not a physical symptom. Ulcers, backache, and cardiac arrhythmias (a,
b, and d) are all physical symptoms associated with burnout.
9. (Answer: d). It is not true that there must be a genuine, tangible threat (d). Individuals may feel
stressed as a result of encountering a real or even a perceived threat. The other statements are true;
stressorsthat is, demands that cannot be readily dealt withfrequently create feelings of tension
and pressure (a, b, and c).
10. (Answer: c). Reward (c), though a very possible effect of a stressful situation, is not identified as a
concrete determining factor because it is too narrow a parameter. Instead, the possibility of reward is
an aspect of the factor we call consequences (d). The other two determinants that help indicate
whether a stress is potentially positive or negative are choice and control (a and b).
11. (Answer: c). An individuals marital status (c) has nothing to do with that persons
professionalism, which is more accurately measured by his or her ethics (a), demeanor (b), and
appearance at work and in the community (d).
12. (Answer: b). Key components of professionalism include having respect for others as well as oneself
and not always holding oneself in highest esteem (b). Individuals who conduct themselves
professionally also avoid gossip, dress respectfully, exhibit good personal values in their behavior, and
accept responsibility (a, c, and d).
13. (Answer: c). All of these are true except that labels must be OSHA/WHMIS labels, not MSDS labels (c).
MSDS stands for Material Safety Data Sheet, and staff members must know where and how to locate
the MSDS sheets at all times and must use precise labeling of secondary containers (a, b, and d).
14. (Answer: b). A key first step is to prepare the environment before you begin (b)that is, choose a
time and place that will lessen the chances for interruptions. This is more crucial than it seems
offhand. Any interruption to the remaining steps may cost you all your successful negotiating
progress because during all stages, emotions can run high. Collecting everyones perceptions may
seem like a likely first step (c), but rather than do this before that first meeting, it is crucial that
everyone hears the others perceptions. In the same way, all should be involved after that in
helping with developing and envisioning a plan that will help everyone win in a significant way (a
and d).
15. (Answer: b). Attainable goals, not sustainable goals (b), are those described as realistic and within in
reach. Other SMART goal characteristics include specificity of goals (a), measurability (c), and
relevance (d).
16. (Answer: d). It is not necessary (and may not be beneficial) to set multiple deadlines for a time-bound
goal (d). The nature of goals that are time bound often dictates that they have only one given deadline,
not multiple deadlines. The other tasks are essential in achieving goals: creating and defining
objectives, evaluating and monitoring personal accomplishments, and recognizing each need by
describing the results of a set goal (a, b, and c).
17. (Answer: b). Monthly analysis (b) should be completed to establish trends, make comparisons of past
months and past years, note immediate changes, and review fees, inventory comparisons, and credit
policies. There are other obligations that must be met daily, quarterly, and yearly (a, c, and d) to
manage the financial status of a practice.
18. (Answer: a). It is not true that a practices efficiency is maximized by assigning each task to the
most highly qualified employee (a). If that were true, the veterinarian would be handling all the roles
normally assigned to veterinary technicians and veterinary assistants. Instead, by diversifying roles,
not only do you distribute tasks wisely among the team, but everyone performs more efficiently
individually as well because trust inspires people to take pride in their contributions. The other
statements are true. Teamwork is essential; each position should have specified tasks (based on
education and legal limits); and a strong personnel management support is key.
19. (Answer: a). Keep items on hand only when needed (a) is true. When referring to inventory control,
one goal should be to keep items on hand only as they are needed and avoid excesses (c);
minimizedo not maximize (b) the expense of keeping supplies in stock. The turnover rate should
be six to eight times per year depending on the productnot four to six (d).
20. (Answer: b). Yearly inventory expense divided by the average cost of the inventory on hand (b) yields
turnover rate. It can also be calculated on the 80:20 rule, whereby 20% of items stocked account for
75% to 85% of the expenditures. The other calculation formulas (a, c, and d) are incorrect.

Das könnte Ihnen auch gefallen